Download as pdf or txt
Download as pdf or txt
You are on page 1of 200

MRCEM Recall contents

1. Recalls MRCEM primary SBA last 1


2. Recalls MRCEM primary SBA June 2021 60
3. Recalls MRCEM primary SBA 2020 Dec 3- 64
4. Recalls MRCEM primary SBA 11th August 2020-115
5. Recalls MRCEM primary SBA 11th August 2020 134
6. Recalls MRCEM primary SBA 20176t 142
7. Recalls MRCEM primary SBA December 2019 146
8. Recalls MRCEM primary SBA June 2019 134
9. Recalls MRCEM primary SBA December 2018 163
10. Recalls MRCEM primary SBA 2-June 2018 167
11. Recalls MRCEM primary SBA 2017 6th dec 173
12. Recalls MRCEM primary SBA 181
1
DR.alshaima Othman + Dr.Rida

1/ scenario pt with pontain haemorage what presentation?


unconsious
Ipsilateral sensory loss
Ipsilateral motor loss
miosis
2
3

2/ Cerebellar infartion Which blood supply?


Posterior inferior cerebellar artery
Anterior cerebral artery
Middle cerebral artery
4

3 / Consunsual reflex wich is affected


Optic tract
Optic nerve
Optic chiasm
5

4/ Sudden vison loss Which artery occluded

Internal carotid Pale ratina

External carotid
Posrer cerebral artery Both eyes vertical line
INTERNAL CAROTID ARTERY > OPHTHALMIC A > CENTRAL ARTERY
5/Pt after surgery he devolp wound dehiscence wich ion responsible
Copper
Selenium
Zinc And vit C + vit A
6

6/ which organ usually accepted by human immune system?


Cornea
Kidney
Heart
liver

7/ Sodium peruside MOA


Nitic oxide production

8/ Pregnant 28 gestation Came with eclampsia Plan for intubation


Which one CI?
Midazolam
Propafole
Thiopental
Ketamine
eitmodate

9/ Fracture of mid shaft of the humerus which nerve injury?


radius
7

10/Iv drug addiction come with infective endocarditis which causative organism?
Staph aureus
Streptococcus pyogen
8

11/Child with dental caries come with infective endocarditis


Streptococcus pyogen
Streptococcus veridance

12/ Virulance factor of staph aureus??


Protein A

13 Pt well without comorbidities came with leg cellulitis he just has tinea pedis
What is the type of infection??
Opportunistic If pt unwell
Conditional If pt is well
Obligate
Colonization
Answer :
Athlete's foot (tinea pedis) is a fungal infection that usually begins between the toes. It
commonly occurs in people whose feet have become very sweaty while confined within tightfitting
shoes. Signs and symptoms of athlete's foot include a scaly rash that usually causes itching,
stinging and burning.

14 /Q about foot drop what is the injured nerve??


Pronial nerve
Fibular nerve
Siatic nerve
Sural nerve
9

15 / First layer of muscles of sole??


Planter artery
Flexor degirorm brevis
Planter vein
10
11

16 /Lung compliance decrease by what of the following??


Empasema
Lung fibrosis

17/Axillary nerve injury what muscle affected??


12

18/ Ulnar nerve interest

19/injury in antecubital fossa large nerve injured .which muscles will affected?
Apponisis pollicus
13

The following 3 questions about diaphragm


14

20\ scenario of diaphragmatic rupture osophegus penetrate diaphragm with


level?
T 10
T8
T12

21\Inferior vena cava penetrate diaphragm wich level?


T8
T10
T12
22\When aorta penetrat the diaphragm T12 what structure pass with it?
Thoracic duct
15

Phrenic
Vagus
16

23/ Xiphesternum dermatome which level ?


T6

24/ Pt with ureter stone pain through which ganglion?

25/Blood supply of Meckel’sdiverticulum?


Superior mesenteric
Inferior mesentric
17

26/Supra colic organs??


Stomach
Illium
Kidney
18
19

27/Lithium toxicity??
20

28/ scenario pt with heart failure wich receptor responsible of his dyspnea?
J receptor

29/Cause of cough in ACE?


Decreas degradation of bradikinin
21

30/Moa of midazolam?
Gaba agonist

31/Moa of lidocaine?
Na chanal block
32\Moa aminotryptan
Serotonin and noradrenalin reaptacke inhipters

32/Brain stem infarction which not affected


Chewing ,
swallowing ,
gastric empty ,

33\what drugs contraindicated on BPH ?


hyocin And all antimuscrinc drugs

34/10 months baby with mucopurulent rash & fever cause?


rubella
22
23

35/Intercranial bleeding with INR 5.8 what to give?


Protamine sulfate Warfarin toxicity

It’s a major bleeds Stop warfarin, give phytomenadione intravenously, give dried prothrombin complex
24

Concentrated prothrombin

36/ scenario pt with femar fracturs and internal bleed what first compensatory
mechanism when dec. BP
In. Contractility
Decrease prephral resistance
Baroreceptor
25

37/Child with sickle, has knee swelling. wat organism


Salmonella

38/Wound infection after 3 days,


prominent cell..
neutrophils
macrophage
39/Ashmatic pt with exacerbating symptoms dysnea and wheeze ,, wat
medication was taken first ?
Salbutamol
Thiophilin
atrovent
40/ Pheocromocytoma which receptor?
Alph1
26

alpha2
beta 1
27

beta2

41/Gluteal pain aggravated when doing hip flexion and adduction which muscle?
Pictenious

42/Course of great saphinous vein ?


anterior to medial maliolus
Posterior to medial maliouls

43/Pregnant 32wk e thrombocytopenia and anemia what the cause?


Autoimmune
HELP syndrome
Gastecional thrompocytopenia
28

B12 defecancy

Gestational thrombocytopenia (defined as a mild thrombocytopenia, occurring during the 3 rd


trimester with spontaneous resolution postpartum and no neonatal thrombocytopenia) is the
most common cause of thrombocytopenia during pregnancy
44/sign of Hypokalemia on ECG?
prolonge QT interval
U wave

45/Tast sensation on anterior 2/3 tongue?


29

vagus
Chorda

46/ scenario pt with paraplegia and urine incontinence?


Central cord syndrome
Anteriour cord syndrom
30

47/ Pernicious anemia Site of parietal cells?


Stomach
dudenum

48/Site of insertion of peroneal longus?


First metatarsal and medial cuniform
31

49/Fracture of medial part of clavicle which muscle affected?


Sternoclidomastoid
Scaleneus
Trapezius
32

50/ which nerve pass in inguinal canal?

Ilioinguinal nerve

inguinal canal
33

51/Incubation period of varicella zoster


14 to 21 weecks
10days 10 -21
days
24 days
52/ child with rash after how many days of appearance of rash became not
infectious?
4 days
7 days
34

53/Enlargment of left atrium compress which organ?


Oesophagus
35

54/ Baby defecate after eating due to which reflex?


Gastrocolic reflex
Oesophagcolic
enterocolic

55/drugs treat tuberclosis on pregnant pt cause convulsion to baby


Isoniazid
rifampicin
Ethambutol
streptomycin
Pyridoxine deficiency may cause seizures in the newborn. Supplemental pyridoxine should, therefore,
be administered to infants on INH or whose mother is taking the drug.
56/pt with shock and metabolic acidosis after given fluid his BPand HR return to
norml but acidosis not corrected with normal anion gap what the cause ?
AKI
decrease perfusion
Normal anion gap casuse : saline

57/pt with diarrhea and vomiting what acid base disturbance?


metabolic acidosis
metabolic alkalosis

In success ‫زيزولاكلا وتباجاو ةدك لاؤس يف‬

58/site of absorption of B12?


distal ilium
36

59/which one exocrine pancreas


glucagon
somatostatin
chymotrypsin
insulin

60/ten year boy with easy brusing and lymphocytosis?


acut lymphoplastic leukemia
vonwilbran disease
idiopathic thrompocytopenia
haemophilia

61/pt with eye trauma during eye examination he loses his consciousness what
cause ?
vagus

62/rash on forhead and upper eye what nerve responsible ?


supratrochliar
supraorbital
37

63/ectopic pregnancy which common site


ampulla

64/herdetory angioedema what treatment ?


38

c1estrase Inhibitors

65/What is the most important anion for calculating anion gap?


NA and K Cation

HCO3 and CL
NA and HCO3
Kand CL

66/Pt head up strait eye held on adduction What nerve affected ?


abducent

67/ scenario pt with eye trauma Paralysis on upward gaze due to ?


Occulomotr pulsy
Inferior rectus entrapment
optic

In maxillary fracture, the orbit floor blows out, and the inferior rectus entrapment leads to
problems in upward gaze. The eye can be injured during compression before the ethmoid bone or
the maxillary sinus fractures. About one third of blow-out fractures have an associated eye injury.
68/Pt unable to do OK sign
Nerve to anterior interrouses
39

69/ pt in endemic area of rabis and hit by bat regardless his vaccination what you
decide to give him?
4 dose vaccine
5 dose vaccine with immunoglobulin
One dose vaccin

70/ Nerve injury in antecubital fossa ?


40

median nerve

71/thoractomy done on posteriolatral incision what mussle injury?


latissmus dorsi
serratus antirior

77/pain from parotid gland referd to ?


Auriculotemporal
Posterior auricular

78/root of finger abduction and flexion ?


C5C6
C7T1
Ulnar C8-T1

79/second heart sound which phase?


isovolmetric relaxation
isovolumetric contraction

80/ achilles tendon rupture what drugs responsible?


quinolon

81/pt with sign of acut gout and he ask for low risk of gi bleed option for
treatment?
41

diclofenac
ibuprofen acut gout ‫ف ديفم م ونا بوتكم‬
naproxcin
‫م اذا هيدن الق برب ينسكوربانال هينات‬
‫لكاشمو بلق لكاشم ودنع‬
82/Muscle of forced inspiration?
Sternocldomastoid
Scalenius
Sratus anterior

83/Muscle of force expiration?


Rectus abdominus
42

84/Muscle support arch of foot


tibials anterior
spring ligment
43
44

85/Recptor for compensation of hypotention brorecptor?


baroreceptor

86/Receptor of irritant
cough? J receptor
Streach receptor

Irritant receptors
45

87/ abdominal truma with Retropertional bleeding from wich organ ?


Duodenum
Kidney
Spleen
liver

88/Partial lesion?
contra lateral inferior quaterntopia

89/Pump of proton pump?


Kcl pum
K/H ATPaa

90/Acute Rheumatoid arthritis marker?


Rhomatoid factor
TNF
IL6
‫ رمحالاب‬cytokines ‫ألس ول‬

91/Relation of direct inguinal herina to hypogastric vessel?


Medial to hypogastric vessels
Latral to hypogastric vessels

92/Tibial nerve sensation of the sole which branch?


Sural

Planter branches
46

93/The nerve supply trapizus exsit the skull through?


jugular foramen

Accessory
94/ pt on dialysis Filtration of urea by?
Simple diffusion
ultrafiltration

95/ Calcium chloride mo in hyperkalemia? Decrease cardiac cell exit ability


Increase k uptacke by cell
Increase cardic muscles excitability
96/What is the site of most bicarbonate absorption in nephron?
Proximal tubule
100/ Ipratropium moa?
Anti muscrenic
101/scenario pt adult with photophopia and headech which organism?
Grame nagetive diplococci N.meningtide If neonate E.Coli
102/ adult pt with symptom of pneumonia
Streptococcus pnemone
103/ nursing home pt with pneumonia wich organism?
Streptococcus pneumonia CAP
Haemophals influanzi
Ligonila The insulin receptor is a transmembrane receptor that is activated
by insulin, IGF-I, IGF-II and belongs to the large class of tyrosine
104/moa of insulin? kinase receptors

105/scenario pt with MI develope bradycardia occulosion on with artery?


47

Rt coronary artery
106/pt with expressive dysphasia legion wich site?
Anterior
Frontal lobe
Paraital obe Middle

Tempora lobe Middle

Occipital lope
107/ another scenario .??. wernicks area
108/ conduction delay on which node to increase ventricular filling?
AS node
AV node
Purkunjy fiber
Pundle of hiss
109/scenario pt cannot hold papers between palm and thump?
Adductor polisis longus Fortman sign ulnar nerve

Abducter previous
110/scenario pt with gastroenteritis develop paralysis?
Campylopactor jijini
111/ Q about aldestron
Aldosterone is a steroid hormone synthesized in and secreted from the outer layer of
the adrenal cortex, the zona glomerulosa.
112/orbicularis occuli supply by which nerve?
Facial nerve Upper by its tempral branch lower by its
zygomatic branch
113/ site of action of thiazide?
Distal tubule NaCL
114/site of action of ADH? Aquapurin

Distal tubule and collecting duct

Loop. > thick ascending


Thiazide early distal. Na/cl
Osmotic. Glomerual lumen
48

Potassium sparing. Distal nephrone Na/K atapase


49

115/ frusemide act on wich


receptor? NA/K/CL cotransporter
116/mannitol acte by?
Osmosis Glomerual lumen

117/ macrophage came from which cell ?


Monocyte
118/scenario pt came from centor of amarica not stable on senario?
Falcibram malaria
118/pt with cholestridum defecalis how to protect ?
Wash by water and soap
119/which move curve to left?
Fetal haemoglobin

120/pt un able to do hand grip?

Median nerve
The answer is : Radial nerve
50

121/the central chemoreceptor sensitive to?


Partial co2 on blood pH of CSF

Ph on blood
122/multiple sclerosing the cause?
Demylanation
Antibody attack receptor
123/effect of parathyroid on calcium?
124/pepsinogen from chif cell were?
Stomach
125/cholycystokinin from I cell were?
51

Duodenum And jijenum


126/which increase bile production?
Cholycystokinin
Glucagon
127/which increase gastric empty?
Sympathetic
Secretin
Gastrin
Type II on oral hypoglycemic
128/pt diabetic you advice to not drive long distance for wich group of peoble?

129/ scenario pt with sweating and hoseness voice ?


Putatory adenoma
130/ which layer after skin and superficial facia on lp?
Supraspinus
131/wich muscls responsible for sholder abduction ?
Supraspinatus
132/ scenario with axillary nerve injury which muscles affected?
52

Teres minor
133/pt with leg cellulitis which organism?
Staph aureas
Streptococcus pyogen
134/pt after recived antibiotic develop skin rash ?
Phenoxymethylpencellin Glanduar fever ??

135/catacholamin release ?
53

Adenal medulla
136/iron absorbtio?
Duodenum and jejenum
137/type II hyper sensitivity due to?

Immunmediated hypersensitivity IgG +IgM


138/which immunoglobulin responsible on type III hypersensitivity?
Igg
54

139/thiophilin intract eith wich drugs cause sizer?


NSAID And quinolone Also cipro with NSAID convulsions

140/pt with hypertensive urgency they want to decrease BP25% what the MAP?
Diastolic+1/3 puls pressure
141/ pt with pulmonary stenosis wich increase?
Afterload
55
Preload
Contractility
142/pt with hypovolemia receiver fluid to increase?
Preload
143/pt running on marathon ATP on first seconds develop from?
Glycolysis
anaerobic from musclse
144/function of golgi apparatus ?
Protin fold
Nuclic acid synthesis
56

145/athalets with hypotension which parameter he have more than normal


person ?
Strok volume
Heart rate
Cardic out put
146/pt alcoholic with hypoglycemia not corrected by glucagon due to ?
Decrease glycogen on liver
147/pt need femora nerve block what istructure immediately lie medial it
femoral nerve?
Femoral artery
148/pt with complete ptosis due to Complete superior tarsal > sympathatic
innervention
?
Oculomotor nerve pulsy
Complete ptosis is due to complete oculomotor nerve palsy. Partial ptosis is due to a
dysfunction of the sympathetic pathway leading to paralysis of Muller muscle.

149/pt with abdominal trauma with paralysis of hemidiaphragm due to ?


Phrenic nerve injury
150/reporalization on cardic cycle due to ?
K efflex
151/sign of hypocalcemia on ecg?
QT prolongation
The ECG hallmark of hypocalcemia remains the prolongation of the QTc interval
because of lengthening of the ST segment, which is directly proportional to the degree
of hypocalcemia or, as otherwise stated, inversely proportional to the serum calcium
level. The exact opposite holds true for hypercalcemia.
152/ muscles responsible of adduction?
Pectenius
153/pre renal parameter?
Urin sodium Low
GFR>20
57

154/ scenario of pt with abdominal pain with history of splenectomy what


organism responsible?
58

Ecoli
Ligonela
155/scalp sensation and upper eye brow? Supraorbital
Supraocular nerbe
156/sensation of upper lip? Infraorbital

Zygomatic
157/pt after RTA develop poly urea what the cause? Diabetes insidous post traumatic
158/ scenario pt with thyroid storm what the initial management?
Propranolol
159/thyroid hormone
///? Increase protein turnover
Increase protein syntheses
160/old man with chlorhydrias what the cell affected?
Parietal cell
161/first cell in inflamatry phase ?
Neutrophil
162/ ASA cause gastric irritation due to?
decrease prostaglandin
163/mechanism of action of NSAID? Inhibit irrecoverable COX
164/muscles attached to latral epichindyl? Posterior forearm ( extensors)

165/ scenario pt with ABG SHOW PCO2 10Kpas?


Pt arrested long time
Pulmonary empolisim PE hypocapnia+Hypoxia respiratory alkalosis

Opid overdose
166/ antiacid intract with absorbtion of wich drugs?
59

Digoxin Phenytoin+isoniazid +chlorobrmazin+tetracycline


Amidaron
167/pt with pulmonary hypertension wich chamber increase pressure can cause
thise?
Rt atrium
Pulmonary vein
Rt ventricle
Left atrium
168/pt with hypotension which is more affect ?
Capillary
Vein
Venuole
Artery
Arteriol
169/influenza virus virulence?
Haemaglutnation
170/mitochondria function?
171/statistic Q
172/statistic Q
173/statistic Q
174/blood supply to ascending colon?
Superior mesenteric artery
175/pt with obstructive lung disease ??\\
Increase FVC
Increase PEV
60

June 2021 recalls Dr. NEHA & DR.RAFI ULLAH

I to inferior epigastric vessels


candidate
1 Direct Inguinal hernia - medial
2 m/s action tibialis anterior - dorsiflexion and inversion

3.medial epicondyilitis- Ulnar nerve- wrist flexors

4. Pregnant lady with eclampsia with high bp safest dx to be choosed- labetolol

5. A 50yr HIV patient; CD4 <80; org- crytococcus; kblesiella; tb

6. Bp falling down; receptors activated: baroreceptors

7 A year old developed cough, scenario of croup - parainfluenza virus.

8.osteomyelitis organism- staph auereus

9.osteomyelitis with sickle cell anaemia - salmonella type.

10. gram +ve diplococci - strep. Pneumonia

11. Rusty nail gram +ve rods- clostridium

12. Measles scenario; safe time to return - 4 days after fever

13. Subscapularis- liftoff test

14. Type 2 DVLa : oral hypoglycaemic drugs with complications , insulin independent

15. Male hematocrit value- 40-49%

16. Cerebellarlesion - ipsilateral palsy

17. Taste of posterior 1/3 of tongue- glossophalyngeal Nerve

18. 8 week old baby vaccinated till date; which vaccination not be given: Rubella 12,months

19. Grave's disease- increase t3

20. Duodenum superior part whichlevel- L1

21. Pneumoperitonum at level of - duodenum

22. Lyme disease causative organism - borrelia budgiformi

23. Popliteal fossa injury- superior lateral border- Biceps femoris

24. Esophagus compressed behind -left atrium

25.bp resistance control by which vessels effected - Arterioles


26. Gastric emptying decreased by - decrease ph duodenum

27. Supra Clavicluar fossa - c3

28. Diaphragm pierces oesophagus level _ T 10

29. Level of nipples- t4

30. Gout patient with Heart failure; htn- colchicine

31. 39yr old on diclofenac has gastric issues - add ppi

32. Anaphylaxis _ histamine

33. Patient developed tingling sensation on upper lip - ace inhibitors _ lisnopril

34. Asthma patient on steroids, which hormones effected most- cortisol/ aldosterone

35. benzodiazepam moa- GABA receptors.

36. Clostridium patient prevention- hand wash with soap.

37. Pontine hemmorhage- reduced level of consciousness , adducted unable to abduct.

38. Pepsinogen secreted by- cheif- cells.

39. Spiral groove- radial Nerve.

40. Ulnar nerve injury- c7/t1

41. Injury to lateral side of knee- commonfibularnerve.

42. Loss of lateral side of heel- sural nerve/ Tibial Nerve.

43. NSAIDs- hemolytic anaemia - mefnanemicacid.

44. Maculopapular rash- due to antibiotic- amoxicillin.

45. Lateral knee compartment action- plantar flexion and - eversion

46. Superficial layer on foot - flexor digitorum brevis.

47. Urea excreted by _ simple diffusion.

48. Catacelomines secreted - adrenal medulla

61

49. A 35year old patient has diarrhoea and vomiting : vasopressin.

50. Infections mononucleosis- lymphocytes.

51. Hemostasis- neutrophils/ platelets.

52. Medial to Nerve (femoral )- femoral artery.

53. Nagma given in fluids normal anion gap- saline administration.

54. Sodium nitroprusside- vasodilator - nitric oxide.

55. Cavernous sinus; medial rotation and adduction - so4.

56. Cavernous sinus; outward & downward - oculomotor nerve.

57. Leptospirosis- invasion of breached skin/ intact skin

58. Giardia - sucking disc.

59. Regional anaesthesia in wrist block- prilocaine|bupivacaine

60. Megaloblastic anemia - nitric oxide.

61. Portal htn with varies - left gastric/ azygousvein.

62. Inferior medial border fracture to the face- maxilla/ethmoid /sphenoid

63. Pulmonary hypertension-blood flow- righatrium / edv/ edp

64. Lying down to supine position - TV/ Rv/FRC.

65. Sob + pulmonaryodema- increases physiological dead space / FRC /tv.

66. After exercise; drinks water- increase in extracellular fluid.

67. Normal v-q ratio- neuromuscular disease.

68. Fever with temp on skin, main regulator- sympathetic activity/ vasoconstriction/vasodilation.

69. S1 heart sound- IVC.

70. Orbicularis oculi- facial nerve.

71. Right side face weakness , sensoryloss - MCA.

72. Pt with hypertension and headache , change in voice , pitutory tumour- bitemporal homonymous
hemianopia.

73.jaw reflex- trigerminal nerve.

74.medial Nerve injury spared which side- little finger.

75. Wrist drop humerus fracture- radial nerve.

76. At wrist injury inthenar emminence- oppenens pollicies.

77. Fromet sign- adductor pollicis

78. Abscess on buttock enlarge lymph node - superior laterally /superior medial.

79. Femoral canal- femoral vein.

80.patellar dislocation- Vastus lateralis/vastus medialis.

81. Extension of hip+ medial rotation- adductor Magnus.

82. Anterior lateral thoracotomy - serratous anterior/ lattismusdorsi.


Q
83. Female with acute onset of confusion; stepping gait after alcohol- Treatment - thaimine

84. Anorexia patient - vitamin d.

85. 50% reduction of diameter rca , which parameter will increase by 16 times - flow /resistance.

86. A patient with blood transfusion developed tingling ,numbness on upperlip and rashes- ty 1/2/3/4

87. When the clavicle fractured (medial) proximal portion- sternocledo mastoid M/s.

88. Increase in complication of central Iv line left - subclavian / oseophagus / thoracic duct/
pneumothorax

89. Hypocalcemia - PTH.

90. Absent cremastric reflex- genitofemoral nerve.

91. Loss of sensation of iliac crest- ilioinguinal/ iliohypogastric

92. Baby 10 day old with purlent discharge- chalymidia.

93. 10 old boy complication purulent discharge _ complication- aponeicbreath /

94. Adduction+ internal rotation- pectoralis major / pectoral minor/ serratous anterior.

95. Pain in right hypo chondrium due to gallbladder : CCK.

96. Nephrotic syndrome- increase capillary wall pressure / oncotic pressure

97. Central cord syndrome - compression - Pca/ compression of ligament flavum/ vertebral artery.

98. Appendicitis - retrocaccal / iliocaecal.

62

99. Abdominal pains bowel ischemia - mc artery - sma/ima/ right colic.

100. Low gastrointestinal effects - ibuprofen.

TILILE
101. Cholestatic Jaundice - co-amoxiclav.

102. In patient with heart failure , tachyponea : j-receptors.

103. After morphine , rr: 5 , Nalaxone compensation : paco2

104. Active Tb : positive mantoux /Zn negative / x Ray consolidation/ x Ray cavitations.

105. Psoas major : T12/ L1.

106. Smoker clear mucous - simple columnar epithelium.

107.Adh stimulation : posterior pituitary / hypothalamus

108. Pancreatic digest fats , stearrrohea : lipase.

109. Hyoscine sideffect - tachycardia / bradycardia / hypotension.

110. Ipatropium bromide: anti muscuranic receptors.

111. 9 month old baby not vaccinated : MENC

112. IRON - absorption : duodenum -

113. VITAMIN B12 : ileum.

114. amytriptyline overdose with wide complex tachycardia - blocks NA / ca / potassium .

115. Sub acute cord syndrome : vitamin B12.

116. Surfactant - phospholipid

117. After intubation supine position increase which volume - TV/cvp

118. Emphysema - increases physiological dead space

119. Pulmonary hypertension- right ventricular load / after load

120.s1 heart sound : closure of mitral valve

121. S2 heart sounds while splitting : pulmonary valve

122. Absolute Refractory period due to : inactivation of na channels / k channels

123. Carboxy hemoglobin - shift to left

124.patient with trauma bp 70/40 which effect - TPR DECREASES

125. Juxtaglomerular apparatus - renin

126.metabolic alkalosis- due to hcl Loss.

127. Renal compensation of metabolic acidosis by - hco3 /ammonia.

128.low aniongap- hypoalbunemia.

129. Polyuria , high osmolality : ADH

130.decrease in na increase in k - aldosterone

131.calcium correction : need albumin

131. Ant pitutary hypofunction : FSH/ LH/ CRH/ prolactin / GH.

132. SIADH : fluoxetine

133.glucagon Acton -liver.

134. Glucagon acts on- alpha-cells.

135.Type 2 dm- increase in resistance to insulin.

136.ecf more in - interstial fluid

137.phenytoin-p50 induces.impaired level function.

138. Hydrocortisone Moa : leukotrine inhibitor on asthma- reduces inflammation.

139.hypokalemia- prominent u wave.

140. Broad qrs- hyperkalemia.

141.loop diuretics -thick ascending limb.

142. Aspirin- inhibition of thromboxane

143. Acyclovir- nuclei acid synthesis inhibitor.

144.right middle zone consolidation - streptococcal pneumonia.

145.meningitis- Ig A protease.

146.whooping cough- notifible disease.

147.optic neuritis - loss of saltatory conduction.

148. Pernicious anaemia due to destruction of cell ; parietal cell

149. Anus loss after trauma - s2 to s4.

150. Clostridium diffcle : cytotoxin / endotoxins.

63

151. Frank starling law- increase contraction -increase volume.

64

2020 Dec 3, MRCEM Primary Q & A


by Alshaima othman

A. Upper Limb
1. Pitcure of Lift- off test- Subscapularis,……

2. Woman age ?55 no history of injury, cannot abduct, her work plasterer—Supraspinatus,…0-15 ….

3. Pt with shoulder dislocation and loss of sensation in rebridgemental area- Axillary Nerve,….

4. H/O injury ( not mention area) Pt can’t abduct above 90 degree and injury from – Roots, Divisions,
65

Trunk, Lateral cord, Medial cord


posterior cord gives Axillary nerve > Trapezius
C5C6C7 Long thoracic > serratus anterior

5. Wrist injury- Choose from position which can’t do- Opposition of thumb, Abd, Add,….

6. After injury, can’t do finger adduction, can do wrist extension, C7-T1, C5-6, C6-7,….

7. Feature of Fromet sign, which muscle— Adductor pollicis,….

8. Antecubital fossa injury and large nerve injury, which function impair least- Little finger ( its with Ulnar
nerve ),…

9. Radial Nerve related….

B. Lower limb
1. Pt with loss of sensation below groin after injury- L1, L2,….= , ilioinguinal external genetilia + upper
medial thigh

2. Pt with loss of sensation over iliac crest- iliohypogastric, ilioinguinal, femoral,….


66

3. Injury and avulsion fracture 5th metatarsal , which will injure- peroneus brevis, peroneus longus,…

4. Muscle of hip flexion- psoas major L2L3L4, Sartorius femoral , pectineus femoral , iliacus femoral all
do that

5. Injury to sole superficially and which will injure- Lateral planter Ar more superfacial, Medial
paternal Ar, Tibialis posterior, FDL,….

6. Nerve injury which pass between medial mellolus and calcaneus ( tarsal tunnel )- Tibia, Saphoneus,
sural, …

7. Lateral border of femoral triangle- Sartorius, adductor,….


67

Roof :fascia lata


Floor: adductor longus pectinous – iliopoas
• Femoral sheet contain all connect EXEPT femoral nerve
Also Lymphatic vessels - draining the deep inguinal lymph nodes.
• Deep lymph node - the lacunar node.
• Empty space.
• Loose connective tissue.
( Google)

8. In femoral hernia, in anatomical proximity, which will be compressed- Femoral vein, AR, Nerve,
Iliacus, Psoas
Femoral hernias invariably compress the femoral vein, while it is rare for an
inguinal hernia to compress the femoral vein. ( Google)
68

9. Testicular torsion and cremestic reflex, which nerve- Genitofemoral, iliohypogastric, ilioinguinal,…

10. Nerve injury when to Adductor canal- Saphenous, Obturator,…


The adductor canal (Hunter’s canal, subsartorial canal) is a narrow conical
tunnel located in the thigh.
It is approximately 15cm long, extending from the apex of the femoral triangle to
the adductor hiatus of the adductor magnus. The canal serves as
a passageway from structures moving between the anterior thigh and posterior
leg.
69

The adductor canal is bordered by muscular structures:


• Anteromedial: Sartorius.
• Lateral: Vastus medialis.
• Posterior: Adductor longus and adductor magnus.
The adductor canal runs from the apex of the femoral triangle to the adductor
hiatus – a gap between the adductor and hamstring attachments of the adductor
magnus muscle

Contents
The adductor canal serves as a passageway for structures moving between the
anterior thigh and posterior leg.
It transmits the femoral artery, femoral vein (posterior to the artery), nerve to the
vastus medialis and the saphenous nerve – the largest cutaneous branch of the
femoral nerve.
As the femoral artery and vein exit the canal, they are called the popliteal
artery and vein respectively.
70
71

11. Sole of foot nerve block which one--……tibial


72

12. Anterior compartment syndrome, when pain elicited—extensor halluces longus, peroneus,….
Pain, paresthesias, and tenderness in both the ischemic muscles and the region supplied by the
deep common fibular nerve are exhibited by patients suffering from this condition. Sensitivity to
passive stretch and active contraction are common, and tend to increase the symptoms.

13. Pt come with direct hernia, where—Medial to inferior hypogastric Ar, Lateral,…..

C. Thorax
1. Thrombosis related and CT, IVC pierce diaphragm- T8, T10,….

2. Oesophagus pierce diaphragm- T10, T8,…

3. Child with swallowing coin and it is in oesophagus, which area impact in anatomical position- T5, T10,
C6,…
ABCD

4. Pt with pneumia, auscatation for middle lobe—Rt 4th rib, Rt 6th rib, lt 4,lt 6,…
73

5. Feature of thoracic outlet syndrome, which vessel- subclavian, brachiocephalic, jugular,…

6. Hyperinflated lungs, which factor affecting or increasing

RV + TLC

D. Head & Neck, CNS


1. IVH only in lateral ventricles and hydrocephalus, which area block—Foramen of Monro also
called interventricular foramen , Cerebral aqueduct,…

The fourth ventricle communicates with the subarachnoid space through the
lateral foramen of Luschka, located near the flocculus of the cerebellum, and
through the median foramen of Magendie, located in the roof of the ventricle.
Most of the CSF outflow passes through the medial foramen
74

2. Basal skull # and loss of sensation in upper teeth and upper lip, which
foramen—Rotundun, Ovale, stylomastoid,…
Maxillary nerve ( infraorbital )

3. Injury in front of lt preauricular, which facial N branch—Temporal, Auricular, ….

4. Nerve to orbicularis oculi- Facial with temporal and zygamotic branches,


Trigeminal, occulo?,….

5. Injury toeye at 12 o clock position 2mm L/W to conjunctiva, which injure—


Tarsal, levator palpebrae, .

6. Middle forehead L/W, which N- Supratrochlear, Supraorbital, …

7. Injury to
N Pt with adducted eye- Abducens ( paralysis of lateral rectus ),
Trochlear, oculomotor,..

8. Horizontal diplopia—Abducens, Trochleal, Occulo,…

9. Injury to superior orbital fissure—Occulomotor, 6, 4,…


75

10. Dental abscess at 2nd molar teeth and Ludgwid angina, which—mylohyoid, jugulohyoid, …..

11. Inferior orbital # which bone—Maxilla, saphenoid, ethmoid,…. ‫اكب واحد‬


‫عشان ر‬

12. Pt with pontine haemorrhage which presentation—Reduced conscious, hemiparesis, Hemiplegia,…


76

13. Headache, profuse sweating, tingling of hands, which visual field defect—Bitemporal hemianopia,
Contralateral hemianopia, monocular loss,…
( pituitary adenoma )

14. Injury to spinal cord, loss of motor lower limb, sensory loss in pain and temperature and preserved
proprioception and fine touch—Anterior cord syndrome, B-S syndrome, posterior cord,…

15. Only injury to proprioception- Posterior cord, Anterior cord,…

16. Protuberance of cervical vertebra—C7, C5, C6,…

17. Ptcan’t do Gag reflex and loss of taste in posterior tonguh—Nerve 9,


7, 5,… Glossopharyngeal nerve
18. Loss of taste sensation in anterior 2/3 of tongue—Nerve 7 Chorda tympani , 5,
9,…
77

19. When LP, after skin and subcutaneous--- Supraspinatus, interspinatus, dura,
pia,…

20. Expressive dysphagia—Frontal lobe, parietal, Temporal ( Wernick ) …


78

21. Cavernous sinus thrombosis, which nerve--- Occulomotor,… Abducent

If lateral : Occulomotor

22. Preauricular Nerve branches……..


79

23. Optic neuritis……..Demyelination

24. One guy look strictly but deviation of eye…..

E. Abdomen
1. Old age, abdominal pain and SMA blockage, which area—jejudonum, colon,…
The superior mesenteric artery (SMA) is a major artery of the abdomen. It arises from
the abdominal aorta, and supplies arterial blood to the organs of the midgut – which spans from
the major duodenal papilla (of the duodenum) to the proximal 2/3 of the transverse colon.

2. Abdominal pain which Artery most—SMA, IMA,Left colic,….

3. Pt with perforation with retroperitoneum --- Duodenum, colon, stomach,…


SAD PUCKER
4. Pernious Anaemia which cell type—Parietal, chief, …..

5. Antibodies in pernious A which organ—Stomach, ileum, colon,…

6. Pain in RHC due to Gall-bladder origin—CCK, Secretin,…

7. In IDA, iron absorption increased— ascorbic acid = Vitamin C+HCL, zinc, milk, Tannin,…

8. H & M in alcoholism, which vessel—left gastric, azygos,periumblical,… >????


80

9. Alcoholic with steatorrhea , which enzyme deficiency—Lipase, trypsin, CCK,…

10. Thrombus occlusion- SMA


SMA At Level L1
mostly occluded in jejunum
81

F. Endocrine
1. 3week old baby vomiting, hypokalaemia, hypochloraemic metabolic alkalosis, which mechanism—HCl
loss, increased HCO3 secretion,….
( Pyloric stenosis )

2. Graves disease, which increased—T3, T4, TSH,..

3. In hypocalcaemia which hormones increased--- PTH, Calcitonin, Vit D,…

4. In hypoglycaemia, inj Glucagon, which wii occur—increased gluconeogenesis, glycogenesis, fatty


acid,…

5. Addison disease, which deficiency—Aldosterone, cortisol,…

6. Glycosuria, which mechanism—Saturation, filtration,…

7. Mineralocorticoid deficiency related

8. Pt come with hypokalaemia, where most absorp of K—PCT, DCT,…

9. Nephrotic syndrome with hypoglycaemia, which mechanism—

[Hyperlipidemia and hyperinsulinemia in kidney diseases occurring with the


nephrotic syndrome]

10. In HD pt, urea mechanism—Simple diffusion, facilitated diffusion, Ultrafiltration

11. In NS infusion, which will change—Increase ECF, ICF,…

12. IN HAGMA Pt, After resus, NAGMA--- HyperCl acidosis of NS, Lactosis acidosis,…

13. Low AG MA, hypoalbuminaemia,….

14. Which compartment is most fluid in ECF--- ISF, IVF,….


82

15. Hypocalcaemia with tetany—Vit D, PTH,…..


Hypocalcemic tetany (HT) is the consequence of severely lowered calcium levels
(<2.0 mmol/l), usually in patients with chronic hypocalcemia. The causal disease
for hypocalcemic tetany is frequently a lack of parathyroid hormone (PTH), (e. g. as
a complication of thyroid surgery) or, rarely, resistance to PTH.

16. Hypothyroid features, ECG changes—Sinus brady, electrical


alternans,…
Common ECG changes in hypothyroidism are bradycardia, low voltage complex, ST
segment depression, QT interval lengthening and increased QT dispersion, flattening
or inversion of T wave, which reflects the prolonged cardiac action potential.
83

The most common ECG changes seen with thyrotoxicosis are: Sinus tachycardia.
Atrial fibrillation with rapid ventricular response. High left-ventricular voltage — i.e
“voltage criteria” for LVH without evidence of LV strain.

17. Posterior pit hormones—Oxytocin, prolactin, ACTH

18. ADH release Posterior pit

19. Man with chronic lung disease with vomiting and diarrhea
Maybe prolonged use of cortisol

20. DM with poor control, which occur—Decreased protein synthesis, increased glycogenesis,
84
85

G. Ht & Lungs, Basic cellular, GI Physio


86

1. ARDS which cell type—Type 2,1,…..

2. Mucocilliary defense, cell type—Columnar epithelial, cuboidal, …

3. Multiple rib fracture and no other injury, which volume affect--- TV, VC, TLC,….All low BUT ratio is high

4. Scuva driving, O2 and Nitrogen which law applied--- Henery, Laplace, Boyle, Charles,…..
Dissolving of gas

5. Ipratrobium bromide nebulized, which mechanism—Vagal related Ach, pul Ach, mast cell,
histamine,…

Ipratropium is a non-selective antagonist of muscarinic receptors which inhibits M1, M2, and M3,

6. 3rd Ht sound—Early ventricular filling, late Ventricular filling, systole,…

7. 4th Ht sound--- Artrial systole, ventricular filling stiff wall ,…….


87

8. ECG elevation aVR depression 2,3,V2 to 6, Ar--- Lt main stem = coronary LCA, LAD, Rt coronary,…
88
89

9. ECG LBBB which Ar affected last 2 month ago—LAD, Rt, circumflex,…

10. Pt with PE and echo, normal RV pressure—20/30systolic , 12-8, 120,….

11. Diasystolic murmur in 5th ICS--- Mitral valve= stenosis , tricuspid, aortic,…

12. In heart failure, which affect--- increased capillary pressure, oncotic pressure,…

13. In shock with resuscitation, which will decreae—CO, SV, TPR,…


90

14. In athletes’, which will increase, SV, CO,…

15. In shock, which distributes--- Arterioles, AR, Veins, capillaries,…

16. In cardiac myocytes, which start AP—Opening of Na, K, Ca,..

17. Excitation-coupling--- Ca, Na,K,…

excitation-contraction coupling (ECC) describes the rapid communication between


electrical events occurring in the plasma membrane of skeletal muscle fibres and
Ca2+ release from the SR, which leads to contraction.

18. Hb F affinity shift to


The left = increase affinity of O

19. Carboxy Hb shift to right—Increased O2, CO2, Reduced??? Shift to the left

20. Pt with metabolic acidosis which will do for compensation--- Carotid body, medulla oblongata,….
Central chemoreceptors ventrolateral medulla > increase RR

21. Pt with shock which will response—Aortic arch, carotid body,…. Baroreceptors in Carotid sinus
>afferent glossopharyngeal + Aortic arch > afferent Vagus

22. Loop diuretics site—Thick ascending limb


91

23. Loop diuretics

24. Anion need to calculate Anion gap—Cl+HCO3Anions, Cl+Na, Cl=K, HCO3=..,…..

25. To correct Calcium,……


92

H.
93

1. HCO3 is contained in which solution---- Hartman, NS, Albumin,…..


Compound Sodium Lactate (Hartmann's) is used: • for intravenous fluid and electrolyte
replacement • as a source of bicarbonate in the treatment of mild to moderate metabolic acidosis
associated with dehydration or associated with potassium deficiency •

I. Pharmacology
1. In thyroid crisis, which drug first—propranolol, carbimazole, steroid,…

2. Pt with AF , Antacid effect which antiarrhythmic--- Digoxin, Amiodarone, Bisoprolol,…

3. Gout with hypertension and IHD, which drugs start—Cochicine, NSAIDS,…

4. Gout and concern for GI effect--- Ibuprofen, diclo, keto, naproxen,..


94

5. Increased INR on Warfarin, which drugs--- Cimeditine, omeprazole, ….

6. Toxicity of Lithium---Ataxia, confusion, tremor,…

7. RA with AKI--- Probenecid, Hydroxychloroquine, NSAIDs,… analgesic induced nephropathy

8. Man with MG, GERD, HT, hyoscine CI—MG, GERD,…

9. Sodium nitripusside MOA—VD,…

10. Amphetamine overdose suspected—Dilated pupils, … ( Sympathomimetic )

non-competitively blocks the reuptake of dopamine and noradrenaline into the terminal by
blocking dopamine transporter (DAT) and noradrenaline transporter (NAT), increasing levels of
dopamine and noradrenaline in the synaptic cleft.

The mechanism of action of amphetamine is complemented by the inhibition of the reuptake


and of monoamine oxidase which acts synergistically to produce a significant increase the
monoamine concentration.

11. Ketamine- Tachycardia, increased salivation, respiratory depression


.tachy_externous muscle spasm+psychotic +HR+BP

12. Girl with rash after taking drug for tonsillitis—Amoxil,. EBV

13. Driver and which antihistamine should he take —Cetirizine, Prochlopropamide, Promethazine,
cyclizine,..
95

14. Acyclovir MOA—Inhibits DNA synthesis Polymerase

15. BP 210 and Dx phaeochromocytoma, which first--- alpha blocker, beta blocker,….

16. Lorazepam in plasma—lipophilic,…. Can cross BBB + less lipid soluble than diazepam + First line in
status epilepticus
Lorazepam is a longer acting, highly lipophilic benzodiazepine compared to midazolam

17. Morphine MOA--- GABA, NMDA,…..


opioid receptors, namely Mu (μ)
Morphine binding to opioid receptors blocks transmission of nociceptive signals, signals pain-
modulating neurons in the spinal cord, and inhibits primary afferent nociceptors to the dorsal horn
sensory projection cells. Morphine has a time to onset of 6-30 minutes
96

I. Patho
1. Wound was opened for 3 days, which stage already finished—Neutrophil leukocytosis, Macrophages,
Vesel genesis,…

2. Chlorhexidine can remove—G positive,..

3. Virulence
97

4. S. pneumonia-- ? phagocytosis and opsonization

5. Peanut allergy- Type 1,2,3,4,5

6. 14 girls with fever, blasts cells—AML, ALL, CLL, CML

7. Nerve gas poisoning

Initial symptoms following exposure to nerve agents (like Sarin) are a runny nose, tightness in
the chest, and constriction of the pupils. Soon after, the victim will have difficulty breathing and will
experience nausea and salivation.

Organophosphorus nerve gases such as sarin, soman and VX act mainly by inhibiting cholinesterase at
cholinergic synapses=Increasing Ach . The consequent accumulation of acetylcholine results in muscle
twitching, glandular hypersecretion and cognitive and mood effects.
98

TREATMENT:
Atropine
peralodoxin

J. Micro
1. Pneumonia with hypoNa—Legionella

2. 82 yrs old woman in nursing home with fever, purulent sputum with endogenous organism—
Pneumocystic jejuni, Legionella, Pseudomonas, Mycoplasma,..

Streptococcus pneumoniae, gram-negative bacilli, Chlamydophila pneumoniae,


and both methicillin-resistant and methicillin-sensitive Staphylococcus aureus
cause most cases of pneumonia in nursing homes.

...
Streptococcus pneumoniae is the most common cause of nursing home–
acquired pneumonia,
__________________________________________________

3. Epiclottitis—Hib

4. Sickle cell A with septic arthritis- Samonella

5. In immunocompromised, which organism

6. Which toxin from Chlotridium difficle—Cytotoxin, endotoxin, Haematotoxin, neurotoxin,..


Spore-forming, Produces Toxin A (enterotoxin) and Toxin B (cytotoxin (
99
100
101

7. Boys with knee pain without injury and lucancy oval in tibia—Staph, Strep, Hib,…

lucency oval tibial bone sign of Osteomyelitis

8. Wound with pus and G + rods—Clostridium perfringes, stap,….

9. Food poisoning—Samonella type 4,.., Shigella

At least 250 different kinds of food poisoning have been documented, but the most
common ones are e. coli, listeria, salmonella, and norovirus, which is commonly called
"stomach flu."
Other less common illnesses that can be transferred from food or food handling are
botulism, campylobacter, vibrio, and shigella.
Samonella after 12 hours
Staph if there is vomiting 1 – 6 hours
E.coli
Staphylococcus aureus (Staph)
• Symptoms begin 30 minutes to 8 hours after exposure: Nausea,
vomiting, stomach cramps. Most people also have diarrhea.

• Common food sources: Foods that are not cooked after handling, such
as sliced meats, puddings, pastries, and sandwiches
Vibrio
• Symptoms begin 2 to 48 hours after exposure: Watery diarrhea,
nausea, stomach cramps, vomiting, fever, chills

• Common food sources: Raw or undercooked shellfish,


particularly oysters
Clostridium perfringens
102

• Symptoms begin 6 to 24 hours after exposure: Diarrhea, stomach


cramps. Usually begins suddenly and lasts for less than 24 hours.
Vomiting and fever are not common.

• Common food sources: Beef or poultry, especially large roasts; gravies;


dried or precooked foods
Salmonella
• Symptoms begin 6 hours to 6 days after exposure: Diarrhea, fever,
stomach cramps, vomiting

• Common food sources: Raw or undercooked chicken, turkey, and


meat; eggs; unpasteurized (raw) milk and juice; raw fruits and vegetables
Other sources: Many animals, including backyard poultry, reptiles and
amphibians, and rodents (pocket pets)
Norovirus
• Symptoms begin 12 to 48 hours after exposure: Diarrhea,
nausea/stomach pain, vomiting

• Common food sources: Leafy greens, fresh fruits, shellfish (such as


oysters), or unsafe water
Other sources: Infected person; touching surfaces that have the virus on
them
Clostridium botulinum (Botulism)
• Symptoms begin 18 to 36 hours after exposure: Double or blurred
vision, drooping eyelids, slurred speech. Difficulty swallowing and
breathing, dry mouth. Muscle weakness and paralysis. Symptoms start
in the head and move down as the illness gets worse.

• Common food sources: Improperly canned or fermented foods, usually


homemade. Prison-made illicit alcohol (pruno).
Campylobacter
• Symptoms begin 2 to 5 days after exposure: Diarrhea (often
bloody), stomach cramps/pain, fever
103

• Common food sources: Raw or undercooked poultry, raw


(unpasteurized) milk, and contaminated water
E. coli (Escherichia coli)
• Symptoms begin 3 to 4 days after exposure: Severe stomach cramps,
diarrhea (often bloody), and vomiting. Around 5–10% of people
diagnosed with E. coli develop a life-threatening health problem.

• Common food sources: Raw or undercooked ground beef, raw


(unpasteurized) milk and juice, raw vegetables (such as lettuce), raw
sprouts, unsafe water
Cyclospora
• Symptoms begin 1 week after exposure: Watery diarrhea, loss of
appetite, and weight loss. Stomach cramps/pain, bloating, increased gas,
nausea, and fatigue.

• Common food sources: Raw fruits or vegetables and herbs


Listeria
• Symptoms begin 1 to 4 weeks after exposure: Pregnant
women usually have a fever and other flu-like symptoms, such as fatigue
and muscle aches. Infections during pregnancy can lead to serious
illness or even death in newborns.
Other people (most often older adults): headache, stiff neck, confusion,
loss of balance, and convulsions in addition to fever and muscle aches.

• Common food sources: Queso fresco and other soft cheeses, raw
sprouts, melons, hot dogs, pâtés, deli meats, smoked seafood, and raw
(unpasteurized) milk
104

10. 3yr old boy with vomiting and diarrhea—Nursing home, last week rice= , fast food,..
Uncooked rice can contain spores of Bacillus cereus, bacteria that can cause food
poisoning. The spores can survive when rice is cooked. If rice is left standing at
room temperature, the spores can grow into bacteria. These bacteria will multiply and
may produce toxins (poisons) that cause vomiting or diarrhoea.

Bacillus cereus is a Gram-positive, rod-shaped, facultatively anaerobic, motile, beta-


hemolytic, spore forming bacterium commonly found in soil and food. The specific
name, cereus, meaning "waxy" in Latin, refers to the appearance of colonies grown on
blood agar.

11. CI of live vaccine in immunocompromised—BCG, VZ, Polio, Pertussis,…


People who are severely immunocompromised include those who: have active
leukaemia or lymphoma, or other generalised malignancy. have received recent
chemotherapy or radiotherapy.
105

12. 8 moth old with rush and vaccine up to date, which may—Rubella, mumps, ….

13. Measles return to school--- 4 Ds after rash

14. Prosthetic heart valve with IE organism S.Epidermis

15. Skin commensal

16. Child with chicken pox when will go to school--- All are crusted, Rash start, swelling,…..

17. 11years come with chest pain(H/O coryzal symptoms)—Coxacie, variecella,….

18. Child come with fever, headache, likely N. meningitis, how will be transmission—Droplet, Airborne,
Faecal-oral,…
106

K. EBM

1. DVLA 2

2. Journal of Head injury in children and plan to apply in your hospital

3. Although woman and man no difference, one is more common in their research—Type 1 error??

4. New medication started

5. Attrition answer
107

2020 Dec 3, MRCEM Primary Q & A

A. Upper Limb

1. Pitcure of Lift- off test- Subscapularis,……

2. Woman age ?55 no history of injury, cannot abduct, her work plasterer—Supraspinatus,…….

3. Pt with shoulder dislocation and loss of sensation in rebridgemental area- Axillary Nerve,….

4. H/O injury ( not mention area) Pt can’t abduct above 90 degree and injury from – Roots, Divisions,
Trunk, Lateral cord, Medial cord

5. Wrist injury- Choose from position which can’t do- Opposition of thumb, Abd, Add,….

6. After injury, can’t do finger adduction, can do wrist extension, C7-T1, C5-6, C6-7,….

7. Feature of Fromet sign, which muscle— Adductor pollicis,….

8. Antecubital fossa injury and large nerve injury, which function impair least- Little finger,…

9. Radial Nerve related….

B. Lower limb

1. Pt with loss of sensation below groin after injury- L1, L2,….

2. Pt with loss of sensation over iliac crest- iliohypogastric, ilioinguinal, femoral,….

3. Injury and avulsion fracture 5th metatarsal , which will injure- peroneus brevis, peroneus longus,…

4. Muscle of hip flexion- psoas major, Sartorius, pectineus, iliacus

5. Injury to sole superficially and which will injure- Lateral planter Ar, Medial paternal Ar, Tibialis
posterior, FDL,….

6. Nerve injury which pass between medial mellolus and calcaneus- Tibia, Saphoneus, sural, …

7. Lateral border of femoral triangle- Sartorius, adductor,….

8. In femoral hernia, in anatomical proximity, which will be compressed- Femoral vein, AR, Nerve,
Iliacus, Psoas

9. Testicular tortion and cremestic reflex, which nerve- Genitofemoral, iliohypogastric, ilioinguinal,…

10. Nerve injury when to Adductor canal- Saphenous, Obturator,…

11. Sole of foot nerve block which one--……


108

12. Anterior compartment syndrome, when pain elicited—extensor halluces longus, peroneus,….

13. Pt come with direct hernia, where—Medial to inferior hypogastric Ar, Lateral,…..

C. Thorax

1. Thrombosis related and CT, IVC pierce diaphragm- T8, T10,….

2. Oesophagus pierce diaphragm- T10, T8,…

3. Child with swallowing coin and it is in oesophagus, which area impact in anatomical position- T5, T10,
C6,…

4. Pt with pneumia, auscatation for middle lobe—Rt 4th rib, Rt 6th rib, lt 4,lt 6,…

5. Feature of thoracic outlet syndrome, which vessel- subclavian, brachiocephalic, jugular,…

6. Hyperinflated lungs, which factor affecting or increasing

D. Head & Neck, CNS

1. IVH only in lateral ventricles and hydrocephalus, which area block—Foramen of Monro, Cerebral
aqueduct,…

2. Basal skull # and loss of sensation in upper teeth and upper lip, which foramen—Rotundun, Ovale,
stylomastoid,…

3. Injury infront of lt preauricular, which facial N branch—Temporal, Auricular, ….

4. Nerve to orbicularis oculi- Facial, Trigeminal, occulo?,….

5. Injury to eye at 12 o clock position 2mm L/W to conjunctiva, which injure—Tarsal, levator palpebrae, .

6. Middle forehead L/W, which N- Supratrochlear, Supraorbital, …

7. Injury to N Pt with adducted eye- Abducens, Trochlear, oculomotor,..

8. Horizontal diplopia—Abducens, Trochleal, Occulo,…

9. Injury to superior orbital fissure—Occulomotor, 6, 4,…

10. Dental abscess at 2nd molar teeth and Ludgwid angina, which—mylohyoid, jugulohyoid, …..

11. Inferior orbital # which bone—Maxilla, saphenoid, ethmoid,….

12. Pt with pontine haemorrhage which presentation—Reduced conscious, hemiparesis, Hemiplegia,…


109

13. Headache, profuse sweating, tingling of hands, which visual field defect—Bitemporal hemianopia,
Contralateral hemianopia, monocular loss,…

14. Injury to spinal cord, loss of motor lower limb, sensory loss in pain and temperature and preserved
proprioception and touch—Anterior cord syndrome, B-S syndrome, posterior cord,…

15. Only injury to proprioception- Posterior cord, Anterior cord,…

16. Protuberance of cervical vertebra—C7, C5, C6,…

17. Pt can’t do Gag reflex and loss of taste in posterior tonguh—Nerve 9, 7, 5,…

18. Loss of taste sensation in anterior 2/3 of tongue—Nerve 7, 5, 9,…

19. When LP, after skin and subcutaneous--- Supraspinatus, interspinatus, dura, pia,…

20. Expressive dysphagia—Frontal lobe, parietal,…

21. Cavernous sinus thrombosis, which nerve--- Occulomotor,….

22. Preauricular Nerve branches……..

23. Optic neuritis……..

24. One guy look strictly but deviation of eye…..

E. Abdomen

1. Old age, abdominal pain and SMA blockage, which area—jejudonum, colon,…

2. Abdominal pain which Ar most—SMA, IMA,Left colic,….

3. Pt with perforation with retroperitoneum--- Duodenum, colon, stomach,…

4. Pernious Anaemia which cell type—Parietal, chief, …..

5. Antibodies in pernious A which organ—Stomach, ileum, colon,…

6. Pain in RHC due to Gall-bladder origin—CCK, Secretin,…

7. In IDA, iron absorption increased—Ascorate, zinc, milk, Tannin,…

8. H & M in alcoholism, which vessel—left gastric, azygos,periumblical,…

9. Alcoholic with steorrhoea , which enzyme deficiency—Lipase, trypsin, CCK,…

10. Thrombus occlusion- SMA


110

F. Endocrine

1. 3week old baby vomiting, hypokalaemia, hypochloraemic metabolic alkalosis, which mechanism—HCl
loss, increased HCO3 secretion,….

2. Graves disease, which increased—T3, T4, TSH,..

3. In hypocalcaemia which hormones increased--- PTH, Calcitonin, Vit D,…

4. In hypoglycaemia, inj Glucagon, which wii occur—increased gluconeogenesis, glycogenesis, fatty


acid,…

5. Addison disease, which deficiency—Aldosterone, cortisol,…

6. Glycosuria, which mechanism—Saturation, filtration,…

7. Mineralocorticoid deficiency related

8. Pt come with hypokalaemia, where most absorp of K—PCT, DCT,…

9. Nephrotic syndrome with hypoglycaemia, which mechanism—

10. In HD pt, urea mechanism—Simple diffusion, facilitated diffusion, …..

11. In NS infusion, which will change—Increase ECF, ICF,…

12. IN HAGMA Pt, After resus, NAGMA--- HyperCl acidosis of NS, Lactosis acidosis,…

13. Low AG MA, hypoalbuminaemia,….

14. Which compartment is most fluid in ECF--- ISF, IVF,….

15. Hypocalcaemia with tetany—Vit D, PTH,…..

16. Hypothyroid features, ECG changes—Sinus brady, electrical alternans,…

17. Posterior pit hormones—Oxytocin, prolactin, ACTH

18. ADH release

19. Man with chronic lung disease with vomiting and diarrhea

20. DM with poor control, whch occur—Decreased protein synthesis, increased glycogenesis,….

G. Ht & Lungs, Basic cellular, GI Physio


111

1. ARDS which cell type—Type 2,1,…..

2. Mucocilliary defense, cell type—Columnar epithelial, cuboidal, …

3. Multiple rib fracture and no other injury, which volume affect--- TV, VC, TLC,….

4. Scuva driving, O2 and Nitrogen which law applied--- Henery, Laplace, Boyle, Charles,…..

5. Ipratrobium bromide nebulized, which mechanism—Vagal related Ach, pul Ach, mast cell,
histamine,…

6. 3rd Ht sound—Early ventricular filling, late Ventricular filling, systole,…

7. 4th Ht sound--- Artrial systole, ventricular filling,…….

8. ECG elevation aVR depression 2,3,V2 to 6, Ar--- Lt main stem, LAD, Rt coronary,…

9. ECG LBBB which Ar affected last 2 month ago—LAD, Rt, circumflex,…

10. Pt with PE and echo, normal RV pressure—20/30, 12-8, 120,….

11. Diasystolic murmur in 5th ICS--- Mitral valve, tricuspid, aortic,…

12. In heart failure, which affect--- increased capillary pressure, oncotic pressure,…

13. In shock with resuscitation, which will decreae—CO, SV, TPR,…

14. In athletes’, which will increase, SV, CO,…

15. In shock, which distributes--- Arterioles, AR, Veins, capillaries,…

16. In cardiac myocytes, which start AP—Opening of Na, K, Ca,..

17. Excitation-coupling--- Ca, Na,K,…

18. Hb F affinity shift to

19. Carboxy Hb shift to right—Increased O2, CO2, Reduced???

20. Pt with metabolic acidosis which will do for compensation--- Carotid body, medulla oblongata,….

21. Pt with shock which will response—Aortic arch, carotid body,….

22. Loop diuretics site—Thick ascending limb

23. Loop diuretics

24. Anion need to calculate Anion gap—Cl+HCO3, Cl+Na, Cl=K, HCO3=..,…..

25. To correct Calcium,……


112

26. HCO3 is contained in which solution---- Hartman, NS, Albumin,…..

H. Pharmacology

1. In thyroid crisis, which drug first—propranolol, carbimazole, steroid,…

2. Pt with AF , Antacid effect which antiarrhythmic--- Digoxin, Amiodarone, Bisoprolol,…

3. Gout with hypertension and IHD, which drugs start—Cochicine, NSAIDS,…

4. Gout and concern for GI effect--- Ibuprofen, diclo, keto, naproxen,..

5. Increased INR on Warfarin, which drugs--- Cimeditine, omeprazole, ….

6. Toxicity of Lithium---Ataxia, confusion, tremor,…

7. RA with AKI--- Probenecid, Hydroxychloroquine, NSAIDs,…

8. Man with MG, GERD, HT, hyoscine CI—MG, GERD,…

9. Sodium nitripusside MOA—VD,…

10. Amphetamine overdose suspected—Dilated pupils, ….

11. Ketamine- Tachycardia, increased salivation, respiratory depression

12. Girl with rash after taking drug for tonsillitis—Amoxil,.

13. Driver and which antihistamine—Cetrizine, Prochlopropamide, Promethazine, cyclizine,..

14. Acyclovir MOA—DNA synthesis

15. BP 210 and Dx phaeochromocytoma, which first--- alpha blocker, beta blocker,….

16. Lorazepam in plasma—lipophilic,….

17. Morphine MOA--- GABA, NMDA,…..

I. Patho

1. Wound was opened for 3 days, which stage already finished—Neutrophil leukocytosis, Macrophages,
Vesel genesis,…

2. Chlohexidine can remove—G positive,..

3. Virulence
113

4. S. pneumonia-- ?phagocytosis and oposonization

5. Peanut allergy- Type 1,2,3,4,5

6. 14 girls with fever, blasts cells—AML, ALL, CLL, CML

7. Nerve gas poisoning

J. Micro

1. Pneumonia with hypoNa—Legionella

2. 82 yrs old woman in nursing home with fever, purulent sputum with endogenous organism—
Pneumocystic jejuni, Legionella, Pseudomonas, Mycoplasma,..

3. Epiclottitis—Hib

4. Sickle cell A with septic arthritis- Samonella

5. In immunocompromised, which organism

6. Which toxin from Chlotridium difficle—Cytotoxin, endotoxin, Haematotoxin, neurotoxin,..

7. Boys with knee pain without injury and lucancy oval in tibia—Stap, Strep, Hib,…

8. Wound with pus and G + rods—Clostridium perfringes, stap,….

9. Food poisoning—Samonella type 4,.., Shigella

10. 3yr old boy with vomiting and diarrhea—Nursing home, last week rice, fast food,..

11. CI og live vaccine in immunocompromised—BCG, VZ, Polio, Pertussis,…

12. 8 mth old with rush and vaccine up to date, which may—Rubella, mumps, ….

13. Measles return to school--- 4 Ds after rash

14. Prosthetic heart valve with IE organism

15. Skin commensal

16. Child with chicken pox when will go to school--- All are crusted, Rash start, swelling,…..

17. 11years come with chest pain(H/O coryzal symptoms)—Coxacie, variecella,….

18. Child come with fever, headache, likely N. meningitis, how will be transmission—Droplet, Airborne,
Faecal-oral,…
114

K. EBM

1. DVLA 2

2. Journal of Head injury in children and plan to apply in your hospital

3. Although woman and man no difference, one is more common in their research—Type 1 error??

4. New medication started

5. Attrition answer
FRCEM 11th AUGEST (JUNE) 115

2020Recall

1-LIPTOSPIROSIS MODE OF PENETERATION>>>.


INTACT SKIN.
2-SCENARION OF ASCENDING PARALYSIS WITH
GITSYMPTOMS>>> CAMPELOPACTER JEJUNI.

3-SCENARIO OF OSTEOMYLITIS CAUSATIVE ORGANISM>> S. AUREUS.

4-SCENARIO OF MIDDLE RT.ZONE PNEUMONIA WITHOUT


OTHERSYMPTOMS IN ADULT>> STREPTOCOCCUS
PNEUMONIA.

5-MODE OF ATTATCHEMENT OF GIRADIA>> SUCKING DISC.

6-STAGE OF BONE HEALING >> CALLUS FORMATION.

7-SCENARIO OF TRAVELLER to NIGERIA WITH


DETERIORATION LEVEL OF CONSIOUSSNESS>>
PLASMODIUM FALCIPARUM.

8-MODE OF ATTATCHEMENT OF INFLUENZA>> HAEMAGGLUTININ.

9-SCENARIO OF GLOMERULONEPHRITIS WHAT TYPE


OFIMMUNOGLUBULIN>> IgG.

10-NSAIDS CAUSE OF ANAEMIA>> MEFANEMIC ACID.


Mefenamic acid can cause
Aplastic anemia (unique NSAIDS can cause aplastic anemia )
Also cause Thrombocytopenia

11-S/E OF KETAMINE>> TACHYCARDIA


Other S/E of ketamine
Increase sympathetic effect
+++ Increase BP

12-ANTISPYCHOTIC RECEPTOR>> DOPAMINERGIC RECEPTOR


116

13-NALOXONE RECEPTOR>> Mu RECEPTOR

14-ANTIBIOTIC CAUSING CHOLESTATIC JAUNDICE>> C0-AMOXICLAVE.

15-PREGNANT WOMEN WITH PERTUSSUS WHAT


ANTIBIOTICGIVEN>> ERYTHROMYCIN.

16-TENDON INJURY WHAT ANTIBIOTIC>> QUINOLONES


117

17-FOETAL HB CAUSE OF SHIFT OF THE CURVE TO LEFT>>


INCREASE BINDING AFFINITY FOR O2.

18-CARBOXYHB EFFECT ON THE OXYHB CURVE>>bb LEFT SHIFT

19-BLOOD SUPPLY OF DESCENDING COLON>> INFERIOR MESENTRIC VIEN

20-LIFT OFF TEST>>. SUBSCAPULARIS


21-ABDUCTION WEAKNESS 0-15 DEGREE>> SUPRASCAPULAR MUSCLE

22-ANOTHER FOR SUPRASCAPULAR MUSCLE.

23-LATERAL FOOT NERVE SUPPLY>> SUPERFICIAL PERONEAL

24-THORAXIC INLET>> SUBCLAVIAN ARTERY

25-MECHANISM OF ACTION OF IPRATROPIUM PROMIDE>>


acetychol.block.(antimuscarinic )

26-CASE OF LOSS OF CONSIOUSSNESS MOST COMMON CAUSE


OFBLEEDING>> PONTINE

27-SECONED HEART SOUND ON ECG>>T-WAVE

28-PRESSURE IN RT VENTRICLE>>15-30

29-TV>> 7ML/KG

30-LOSS OF CONCENTRATION TO OBJECT WHICH MUSCLE>>


CILIARY MUSCLE

31-START OF WOUND HEALING>>??

32-LUNG VOLUME AFFECTED IN PREGNANCY>>TV

33-THUMB DERMATOME>> C6

34-SECENARIO OF LONG STANDING CORTICOSTERIODS THERAPY


WITH HYPERKALEMIA due to >>DECREASE ALDOSTERONE
118

35-SCENARIO OF PNEUMONIA WITH HYPONATREMIA>>


LIGONELLA PNEUMONIA

36-NIPPLE DERMATOME>> T4

37-UMBLICAL DERMATOME>> T10

38-RASHES AFTER ANTIBIOTICS>> AMOXICILLIN ( glandular fever)

39-ECG WITH ANTERIOR MI>> LAD ‫مكرر‬

40-ENZYME IN PANCREATIC SECRITION NOT INHIBETE


BYPANCREATITIS>> CCK

41-HOMONOMUS HEMIANOPIA WITH MACULAR SPARING


ARTERYAFFECTED>> POSTERIOR CEREBRAL ARTERY.

42-TEMPORAL LOBE LESION>> CONTRALATERAL


SUPERIORQUADRANTANOPIA

43- LP . "2nd gine" BEFOR SUBARACHENOID>> DURA MATTER

44-RABIES VACCINES IN HIGH VIRULENCE AREA >>IMMUNOGLOBULIN +5


DOSES RABIES 1MONTH INTERVAL.( 0.3.7.14.last one bet28-30)

45-LIVE VACCINE>> BCG

46-ASI ATTATCHEMENT>> SARTORIOUS

46- FLEXOR CARPAI RADIALIS QUESTION related ???

48-CASE WITH MEDIAN NERVE SUPPLY muscle affected >> OPPONENS


POLLICIS

49-ANAESTHESIA CAUSNG VASODILATATION>> NITOUS OXIDE (NO)

50-PORTOSYSTEMIC ANASTMOSIS >>UMBLICAL AREA

51-SURFACTANT PRODUCTION>> TYPE 2 PNEUMOCYTE

52-GAS EXCHANGE>>TYPE 1 SURFACTANT


119

53-ANKLE JERK>> L5,S1. more specific (S1 )

54-MECHANISM OF GERD>> LOW HCO3. treatment antiacid

55-AUSCULTATION AT 5TH ICS>> RT MIDDLE LOBE LUNG

56-SCENARIO OF GIRL WITH EPISTAXIS HOW TO ASSESS>> APTT

57-SCENARIO PE >> INCREASE PHYSIOLOGICAL DEAD SPACE

58-STETAORRHEA ENZYME>> pancreatic LIPASE

59-MECHANISM OF GASTRITIS>> INCREASE GASTRIN

60-5 CM ABOVE MEDIAL EPICONDYLE STRUCTURE INJURED>>


ULNARCOLLATERAL ARTERY

61-INGESTION OF COIN LODGED IN OESOPHAGUS>> AORTIC ARCH

62-COMPLEMENT AFFECT>>> STREPTOCOCCUS PNEUMONIA

63-SCENARIO OF SIADH

64-SITE OF ACTION OF THIAZIDE DIURETICS>> DCT

65-GLUCOS ABSORPTION SITE IN THE KIDNEY>> PCT

66-Q ABOUT J-RECEPTOR ??

67-CELLA TURCICA ass with bone # >> SPHENOIDAL

68- Manufacturing of IMMUNOGLUBULIN inside the body >> INNATE


IMMUNITY

69-FLUCLOXACILLIN>> OSTEOMYLITIS AMOXICILLIN NOT AN OPTION

70-Q ABOUT CIPROFLOXACILLIN

71-GOUT WITH HEART FAILURE>> CHOLCHICINE

72-THYROTOXICOSIS TREATMENT>> PROPRANOLOL

73-SCENARIO OF URITHERITIS>> HSV TYPE2(Right answer)


Other options . (SHIGELLA) I didn't remember that
120

74-HEART MURMER WITH VALVE LESION,


CUASATIVEORGANISM>>> STREPT VIRIDANS.

75-PSUDOMONAS TRANSMESSION>> FAN HEATER

76-SLOW DOWN CONDUCTION>> AVN

77-HORMONE CAUSE HYPERTHYROIDYSM ??? TSH

78-FRANK STARLING LOW EFFECT>> DECREASE CONTRACTILITY

79-SCENARIO OF ALCOHOL INTOXICATION WHAT DEFECT >> THIAMINE


DEFICIENCY

80-DESSOLVED GASES>> HENRY LAW

81-GEL USE WITH WHAT TYPE OF ORGANISM>> GM+VE

82-SYSTYMIC VASCULAR RESISTANCE>>ARTERIOLE.

83-MODE OF TRANSMESSION OF NESSERIA MENENGITIDIS>> Respiratory


DROPLET

84-80 YEARS BACK PAIN ESPECIALLY LUMBAR AREA WITHOUT


EVEDENCEOF TRAUMA. >>INCREASE ALKALINE PHOSPHATASE ALMOST
MULTIPLEMYELOMA

85-MACROPHAGE ORIGIN. >>MONOCYTES

86-COMPENSATION FROM METABOLIC ACIDOSIS WHICH AREA


OFBRAIN. >>MEDULLA OBLONGATA

87-COMMENSAL ORGANISM. >>STREPTOCOCCAL PNEUMONIA

88-MACROCYTIC ANAEMIA>>MEGALOBLASTIC ANAEMIA

89-UREA>> SIMPLE DIFFUSION

90-LOW ANION GAP>> HYPOALBUMENEMIA

91-SCENARIO OF SICKLE CELL DISEASE + Osteomyelitis WHAT


CAUSATIVE ORGANISM ASSOCIATED>> SALMONELLA
121

92-PERICARDIOCENTESIS WHICH AREA OF HERAT MAY INHUERED>>


Rt VENTRICLE

93-WHY SODIUM REMAINING HIGH>> ACTIVE TRANSPORT

94-CHIEF CELL SECRETION>> PEPSINOGEN

95-CAUSE OF DELAY GASTRIC EMPTY>> LOW DOUDENAL PH

96-MYOCARDITIS WHICH ORGANISM>> COCXAKI VIRUS

97-COMMON ORGANISM WITH HIV PATIENT


INSCENARIO>> PNEUMOCYSTITIS

98-PAROTID GLAND INJURY WHICH STRUCTURE AFFECTED>>FACIAL NERVE


+ MAXILLARY ARTERY

99-LAPLAS LAW>> INCREASE RADIUS DOUBLE Result will


increase ????? 16 TIMES

100-TYPE 1 DM>> protein KINASE(. Tyrosine kinase !!!! )

101-NERVE BLOCK FOREHEAD TO THE VERTEX>>SUPRAORBITAL N

102-question about parietal cell secretion>> HCL

103-addisons>> mineralocorticoids

104-patient with HAGMA corrected with NACL become NAGM what


thecause>> LACTIC ACIDOSIS

105-mechanism of action of hypertonic saline>>increase extracellular fluid

106-septic shock >> alpha 1 agonist

107-question about trimethoprim

108-MECHANISM OF ULCER>> DECREASE PROSTAGLANDIN

109-PATIENT ON RAMIPRIL WITH COUGH CAUSE>> INCREASE


BRADYKININ
122

110-PHEOCHROMOCYTOMA WHICH HORMONE


RESPONSUBLE>> EPINEPHRINE

111-ANT. SHOULDER DISLOCATION which ligament tear >>


GLENOHUMORAL

112-CARDIAC EFFECT OF DECLOFENAC>. > MYOCARDIAL ISCHEMIA

113-IN TYPE 2 DM HYPERGLYCEMIA WITH GLUCOSURIA reason for


appearance of glucose in urine. >> STAURATION IN PCT

114-LUDWIGS ANGINA LYMPH DRAINAGE>> SUBMANDIBULAR

115-SCENARIO RELATID TO OESOPHAGEAL OPENING>> T10

116-FRACTURE SHAFT HUMERUS>>RADIAL NERVE

117-FLEXOR TENDOR TENDON extend to >> LITTLE FINGER

118-ACCESSORY NERVE INJURY>> TRAPEZIUS

119-PICA SCENARIO

120-HUSSELBACH EQUATION WHICH REQUIRED>> HCO3+CO2

121-PREGNANT LADY which lung volume increase >> TV

122-SOB+NORMAL A-a RATION>>neuromuscular disorder

123- ecg changes with hypokalemia>> T wave inversion

124- B12 DEFF.>> ILIAL RESCTION

125-INTENTIONAL TREMOR>> NIGROSTRIATE AREA

126-MECHANISM OF ACTION OF ACYCLOVIR>> INHIBIT DNA

127-BICARBONATE RICH SOLUTION>> HARTMAN”S SOLUTION

128-MECHANISM OF ACTION OF CHOLCHECINE>>


MICROTUBULE POLYMERIZATION

129-DENTAL ABSCESS WITH SOB. Infections spread for which space >>
PARAPHARYNGEAL ABSCESS
123

130-TRAGUS>> AURICULOTEMPORAL NERVE. ‫اخيرا جات‬

131- DELAY IN ELECTRICAL IMPULSE RELATED TO. AVN.

132-QUESTION ABOUT ENDOPLASMIC RETICULUM

133-UNABLE TO FLEX BIG TOE AND FOOT >> FLEXOR HALLUCIS LONGUS

134-HYPERTONIC SALINE>>I NCREASE EXTRACELLULAR FLUID VOLUME

135-QUESTION ABOUT FIRST LINE GIVEN IN ANAPHYLAXIS>> ADRENALINE

136-CENTRAL SLIP DEFORMITY>> BOUTUNIERE

137-FUNCTION OF EXTERNAL INTERCOSTAL MUSCLE>> ELEVATION OF


THERIB

138-EBM ACCORDING NOMBER>> DISCRETE

139-WINGED SCAPUL WHICH NERVE>> LTN

140-INABILITY TO EVERT OR INVERT (CANT REMEMBER) AND PLANTER


FLEXOR ANKLE WITH SEVERE PAIN>> options
TIBIALIS POST
peroneus longus

141-ELBOW EXTENSION DERMATOME>> C7

142-AFTER TRACHEOSTOMY WHAT CAUSING BLEEDING (THEROIDA


EMAARTERY NOT IN CHOICES)>> INFERIOR THROID VIEN

143-BLOOD SUPPLY OF LEFT VENTRICLE>> LCA

144-OSMOLALITY MEASURE>> UREA

145-COPD LUNG VOLUMES QUESTION which one increase???? ‫مكرر‬

146- 1st MOLAR TEETH LYMPH NODE>>SUBMANDIBULAR


147-GAS ATTACK>>REMAINING SIDE EFFECT>> EXCESSVE SALIVATION
ANDLACRIMATION
124

148-WITH DORSAL THORACOTOMY WHICH NERVE AFFECTED>>I

149-ERBS PALSY>>DON’T REMEMBER THE QUESTION

150-HYPOGLYCEMIA WITH NEPHROTIC SYNDROME >>WHAT SITE OFKIDNEY


AFFECTED>> PCT

151>>SCENARIO ABOUT RT TO LEFT SHUNT A - a gradient mismatch

152---156 EBM QUESTIONS >> ICANT REMEMBER

157-HEPATITIS B DIAGNOSE>> HBsAG

158-SCUPA DIVING QUESTION??


s
159 Cerebral edema effect- Donnan effect
160. Prolongation of QT - phase 2
161. Physiological dead space increased in- ? Pulmonary embolism
162. Aortic stenosis ?? -decreased SV/ increased EDV

163. Sensation over right maxilla- infraorbital nerve


164 SAH- ACA

165. Corneal abrasion-? Long ciliary nerve

166. QRS-IVC

167. Gram negative organisms- cephalosporins

168. Left pupil dilates when passing light from right to left side cons. consensual reflex by 3rd CN

‫مع خالص االماني بالتوفيق والنجااااح‬

* DOLAAA *
125

FRCEM 11TH AUGUST 2020

1-LIPTOSPIROSIS MODE OF PENETERATION>>> BREACHED SKIN.

2-SCENARION OF ASCENDING PARALYSIS WITH GIT


SYMPTOMS>>>CAMPELOPACTER JEJUNI.

3-SCENARIO OF OSTEOMYLITIS CAUSATIVE ORGANISM>> STAPH. AUREUS.

4-SCENARIO OF MIDDLE RT.ZONE PNEUMONIA WITHOUT OTHER


SYMPTOMS IN ADULT>>STREPTOCOCCUS PNEUMONIA.

5-MODE OF ATTATCHEMENT OF GIRADIA>>SUCKING DISC.

6-STAGE OF BONE HEALING >>CALLUS FORMATION.

7-SCENARIO OF TRAVELLER WITH DETERIORATION OF


CONSIOUSSNESS>>PLASMODIUM FALCIPARUM.

8-MODE OF ATTATCHEMENT OF INFLUENZA>>HAEMAGGLUTININ.

9-SCENARIO OF GLOMERULONEPHRITIS WHAT TYPE OF


IMMUNOGLUBULIN>>IgG.

10-CAUSE OF ANAEMIA>>MEFANEMIC ACID.

11-SE OF KETAMINE>>TACHYCARDIA

12-ANTISPYCHOTIC RECEPTOR>>D RECEPTOR

13-NALOXONE RECEPTOR>>MU RECEPTOR

14-ANTIBIOTIC CAUSING CHOLESTATIC JAUNDICE>>C0-AMOXICLAVE.

15-PREGNANT WOMEN WITH PERTUSSUS WHAT ANTIBIOTIC


GIVEN>>CLARITHROMYCIN.

16-TENDON INJURY WHAT ANTIBIOTIC>>QUINOLONES


126

17-FOETAL HB CAUSE OF SHIFT OF THE CURVE TO LEFT>>INCREASE


AFFINITY FOR O2.

18-CARBOXYHB EFFECT ON THE OXYHB CURVE>>LEFT SHIFT

19-BLOOD SUPPLY OF DESCENDING COLON>>INFERIOR MESENTRIC VIEN

20-LEFT OFF TEST>>TEST OF INFRASPINATUS MUSCLE.

21-ABDUCTION WEAKNESS 0-15 DEGREE>>SUPRASCAPULAR MUSCLE

22-ANOTHER FOR SUPRASCAPULAR MUSCLE.

23-LATERAL FOOT NERVE SUPPLY>>SUPERFICIAL PERONEAL

24-THORAXIC INLET>>SUBCLAVIAN ARTERY

25-MECHANISM OF ACTION OF IPRATROPIUM PROMIDE>>acetychol.


block.

26-CASE OF LOSS OF CONSIOUSSNESS MOST COMMON CAUSE OF


BLEEDING>>PONTINE

27-SECONED HEART SOUND ON ECG>>T-WAVE

28-PRESSURE IN RT VENTRICLE>>15-30

29-TV>>7ML/KG

30-LOSS OF CONCENTRATION TO OBJECT WHICH MUSCLE>>CILIARY


MUSCLE

31-START OF WOUND HEALING>>??

32-LUNG VOLUME AFFECTED IN PREGNANCY>>TV

33-THUMB DERMATOME>>C6

34-SECENARIO OF LONG STANDING CORTICOSTERIODS THERAPY WITH


HYPERKALEMIA>>DECREASE ALDOSTERONE
127

35-SCENARIO OF PNEUMONIA WITH HYPONATREMIA>>LIGONELLA


PNEUMONIA

36-NIPPLE DERMATOME>>T4

37-UMBLICAL DERMATOME>>T10

38-RASHES AFTER ANTIBIOTICS>>AMOXICILLIN

39-ECG WITH ANTERIOR MI>>LAD

40-ENZYME IN PANCREATIC SECRITION NOT INHIBETE BY


PANCREATITIS>>CCK

41-HOMONOMUS HEMIANOPIA WITH MACULAR SPARING ARTERY


AFFECTED>>POSTERIOR CEREBRAL ARTERY.

42-TEMPORAL LOBE LESION>>CONTRALATERAL SUPERIOR


QUADRANTANOPIA

43-GIVE BEFOR SUBARACHENOID>>DURA MATTER

44-RABIES VACCINES IN HIGH VIRULENCE AREA >>IMMUNOGLOBULIN +5


DOSES RABIES 1MONTH INTERVAL.

45-LIVE VACCINE>>BCG

46-ASI ATTATCHEMENT>>SARTORIOUS

47-FLEXOR CARPI RADIALIS

48-CASE WITH MEDIAN NERVE SUPPLY>>OPPONENS POLLICIS BREVIS

49-ANAESTHESIA CAUSNG VASODILATATION>>NITOUS OXIDE

50-PORTOSYSTEMIC ANASTMOSIS >>UMBLICAL AREA

51-SURFACTANT PRODUCTION>>TYPE 2 PNEUMOCYTE

52-GAS EXCHANGE>>TYPE 1 SURFACTANT


128

53-ANKLE JERK>>L5,S1

54-MECHANISM OF GERD>>LOW HCO3

55-AUSCULTATION AT 5TH ICS>>RT MIDDLE LOBE LUNG

56-SCENARIO OF GIRL WITH EPISTAXIS HOW TO ASSESS>>APTT

57-SCENARIO PE >> INCREASE PHYSIOLOGICAL DEAD SPACE

58-STETAORRHEA ENZYME>>LIPASE

59-MECHANISM OF GASTRITIS>>INCREASE GASTRIN

60-5 CM ABOVE MEDIAL EPICONDYLE STRUCTURE INJURED>>ULNAR


COLLATERAL ARTERY

61-INGESTION OF COIN LODGED IN OESOPHAGUS>>AORTIC ARCH

62-COMPLEMENT AFFECT>>>STREPTOCOCCUS PNEUMONIA

63-SCENARIO OF SIADH

64-SITE OF ACTION OF THIAZIDE DIURETICS>>DCT

65-GLUCOS ABSORPTION SITE IN THE KIDNEY>>PCT

66-Q ABOUT J-RECEPTOR

67-CELLA TURCIA>>SPHENOIDAL

68-IMMUNOGLUBULIN>>INNATE IMMUNITY

69-FLUCLOXACILLIN>>OSTEOMYLITIS AMOXICILLIN NOT AN OPTION

70-Q ABOUT CIPROFLOXACILLIN

71-GOUT WITH HEART FAILURE>>CHOLCHICINE

72-THYROTOXICOSIS TREATMENT>>PROPRANOLOL

73-SCENARIO OF URITHERITIS>>SHIGELLA
129

74-HEART MURMER WITH VALVE LESION, CUASATIVE


ORGANISM>>>STREPT VIRIDANS.

75-PSUDOMONAS TRANSMESSION>>FAN HEATER

76-SLOW DOWN CONDUCTION>>AVN

77-HORMONE CAUSE HYPERTHYROIDYSM

78-FRANK STARLING LOW EFFECT>>DECREASE CONTRACTILITY

79-SCENARIO OF ALCOHOL INTOXICATION WHAT DEFFE>>THIAMINE

80-DESSOLVED GASES>>HENRY LAW

81-GEL USE WITH WHAT TYPE OF ORGANISM>>GM+VE

82-SYSTYMIC VASCULAR RESISTANCE>>ARTERIOLE.

83-MODE OF TRANSMESSION OF NESSERIA MENENGITIDIS>>DROPLET

84-80 YEARS BACK PAIN ESPECIALLY LUMBAR AREA WITHOUT EVEDENCE


OF TRAUMA>>INCREASE ALKALINE PHOSPHATASE ALMOST MULTIPLE
MYELOMA

85-MACROPHAGE ORIGIN>>MONOCYTES

86-COMPENSATION FROM METABOLIC ACIDOSIS WHICH AREA OF


BRAIN>>MEDULLA OBLONGATA

87-COMMENSAL ORGANISM>>STREPTOCOCCAL PNEUMONIA

88-MACROCYTIC ANAEMIA>>MEGALOBLASTIC ANAEMIA

89-UREA>>SIMPLE DIFFUSION

90-LOW ANION GAP>>HYPOALBUMENEMIA

91-SCENARIO OF SICKLE CELL DISEASE WHAT CAUSATIVE ORGANISM


ASSOCIATED>>SALMONELLA
130

92-PERICARDIOCENTESIS WHICH AREA OF HERAT MAY INHUERED>>RT


VENTRICLE

93-WHY SODIUM REMAINING HIGH>>ACTIVE TRANSPORT

94-CHIEF CELL SECRETION>>PEPSINOGEN

95-CAUSE OF DELAY GASTRIC EMPTY>>LOW DOUDENAL PH

96-MYOCARDITIS WHICH ORGANISM>>COCXAKI VIRUS

97-COMMON ORGANISM WITH HIV PATIENT IN


SCENARIO>>PNEUMOCYSTITIS

98-PAROTID GLAND INJURY WHICH STRUCTURE AFFECTED>>FACIAL NERVE


AND MAXILLARY ARTERY

99-LAPLAS LAW>>INCREASE RADIUS DOUBLE 16 TIMES INCREASE


TRANSPORT

100-TYPE 1 DM>>PROTIEN KINASE

101-NERVE BLOCK FOREHEAD TO THE VERTEX>>SUPRAORBITAL N

102-question about parietal cell secretion>>HCL

103-addisons>>mineralocorticoids

104-patient with HAGMA corrected with NACL become NAGM what the
cause>>LACTIC ACIDOSIS

105-mechanism of action of hypertonic saline>>increase extracellular fluid

106-septic shock>>alpha 1 agonist

107-question about trimethoprim

108-MECHANISM OF ULCER>>DECREASE PROSTAGLANDIN

109-PATIENT ON RAMIPRIL WITH COUGH CAUSE>>INCREASE BRADYKININ


131

110-PHEOCHROMOCYTOMA WHICH HORMONE


RESPONSUBLE>>EPINEPHRINE

111-ANT. SHOULDER DISLOCATION>>GLENOHUMORAL

112-CARDIAC EFFECT OF DECLOFENAC>>MYOCARDIAL ISCHEMIA

113-IN TYPE 2 DM HYPERGLYCEMIA WITH GLUCOSURIA>>STAURATION IN


PCT

114-LUDWIGS ANGINA LYMPH DRAINAGE>>SUBMANDIBULAR

115-SCENARIO RELATID TO OESOPHAGEAL OPENING>>T10

116-FRACTURE SHAFT HUMERUS>>RADIAL NERVE

117-FLEXOR TENDOR TENDON>>LITTLE FINGER

118-ACCESSORY NERVE INJURY>>TRAPEZIUS

119-PICA SCENARIO

120-HUSSELBACH EQUATION WHICH REQUIRED>>HCO3+CO2

121-PREGNANT LADY>>TV

122-SOB+NORMAL A-a RATION>>neuromuscular disorder

123-ecg changes with hypokalemia>>T wave inversion

124-B12 DEFF.>>ILIAL RESCTION

125-INTENTIONAL TREMOR>>NIGROSTRIATE AREA

126-MECHANISM OF ACTION OF ACYCLOVIR>>INHIBIT DNA

127-BICARBONATE RICH SOLUTION>>HARTMAN”S SOLUTION

128-MECHANISM OF ACTION OF CHOLCHECINE>>MICROTUBULE


POLYMERIZATION

129-DENTAL ABSCESS WITH SOB>>PARAPHARYNGEAL ABSCESS


132

130-TRAGUS>>AURICULOTEMPORAL NERVE

131-ELECTRICAL IMPULSE RELATED TO AVN.

132-QUESTION ABOUT ENDOPLASMIC RETICULUM

133-UNABLE TO FLEX BIG TOE AND FOOT >>FLEXOR HALLUCIS LONGUS

134-HYPERTONIC SALINE>>INCREASE EXTRACELLULAR FLUID VOLUME

135-QUESTION ABOUT FIRST LINE GIVEN IN ANAPHYLAXIS>>ADRENALINE

136-CENTRAL SLIP DEFORMITY>>BOUTUNIERE

137-FUNCTION OF EXTERNAL INTERCOSTAL MUSCLE>>ELEVATION OF THE


RIB

138-EBM ACCORDING NOMBER>>DISCRETE

139-WINGED SCAPUL WHICH NERVE>>LTN

140-INABILITY TO EVERT OR INVERT (CANT REMEMBER) AND PLANTER FLEX


THE ANKLE WITH SEVERE PAIN>>TIBIALIS POST OR PERONEUS LONGUS

141-ELBOW EXTENSION DERMATOME>>C7

142-AFTER TRACHEOSTOMY WHAT CAUSING BLEEDING (THEROIDA EMA


ARTERY NOT IN CHOICES)>>INFERIOR THROID VIEN

143-BLOOD SUPPLY OF LEFT VENTRICLE>>LCA

144-OSMOLALITY MEASURE>>UREA

145-COPD LUNG VOLUMES QUESTION

146-MOLAR TEETH LYMPH NODE>>SUBMANDIBULAR


133

147-GAS ATTACK>>REMAINING SIDE EFFECT>>EXCESSVE SALIVATION AND


LACRIMATION

148-WITH THORACOTOMY WHICH NERVE AFFECTED>>I CHOOSED ANTER.


THORACIX (INTERCOSTAL OTHER OPTION)

149-ERBS PALSY>>DON’T REMEMBER THE QUESTION

150-HYPOGLYCEMIA WITH NEPHROTIC SYNDROME >>WHAT SITE OF


KIDNEY AFFECTED>>PCT??

151>>SCENARIO ABOUT RT TO LEFT SHUNT

152---156 EBM QUESTIONS >>CANT REMEMBER

157-HEPATITIS B DIAGNOSE>>HBsAG

158-SCUPA DIVING QUESTION??


134

Fr
cem pr
imar
yrecal
ls20176thdec

Compi
ledbyDrsar
madal
i.

1.Adr
enal
inedosei
ncar
diacshockact
oacl
s=1:
10k

2.Oesophaguspi
erceatwhi
chdi
aphr
agm l
evel=T10

3.Cont
ent
sofr
ect
ussheat
h=py
rami
dal
is

4.Abdominali
njur
yscenar
io,
cont
ent
sofr
ect
ussheat
h=agai
n
pyr
amidali
s

5.Sl
eab=ANA

6.Thenari
njur
ygl
asspi
eceset
c=opponenspol
ices/
fpl
/fpbet
c

7.Si
adhf
eat
ures=hy
ponat
remi
a

8.Dr
ugcausi
ngsi
adh=f
louxet
ine

9.Waddl
i
nggai
t=supgl
utealner
ve

10. Cr
ushi
njur
y,dr
ugci=suxamet
honi
um

11. Anaphy
laxi
sreact
ionab=I
gE

12. Sur
fact
antf
unct
ion=r
educet
hesur
facet
ensi
on

13. Sur
fact
antf
ormedby=pneumocy
test
ype2

14. Doxy
cycl
ineusei
npr
egnancy=s.
etoot
hdi
scol
our
ati
oni
nbaby

15. Rubel
lamostdanger
ous=i
stt
ri
mest
er(
organogensi
sper
iod)

16. Cr
prel
easedst
imul
atedby=I
L-6

17. Ecgchangesi
nhy
per
kel
eimi
cptbef
oret
hecar
diacar
rest
=tal
l
t
/broadqr
s
135

18. Ret
eroper
it
onealhemat
oma=duodenum (
sadpucker
)

19. Her
niamedi
alt
oepi
gast
ri
cvessel
s=di
recti
ngui
nalher
nia

20. Adenosi
neci=ast
hma

21. L.
phi
ghestsaf
estl
eveli
nadul
ts=l
3-l
4

22. Duodenali
njur
y,d1i
sat=L1

23. Dr
ugt
hati
nct
heact
ionofami
nophy
lenei
ncopd/
ast
hma=FQ:
cipr
o

24. Nonsedat
ingant
ihi
stami
ne=f
enoxi
fi
nadi
ne

25. Ptwi
thepsneckst
if
fness=ant
iemt
icuse=Met
achl
oropr
omi
de

26. Fuer
osomi
deMoa=t
hickascendi
ngl
oop

27. Fuer
osomi
demoa=na/
k/2cl

28. Thi
azi
demoa=dct

29. Car
oti
dsi
nusmassage=atwhi
chl
evel=t
hyr
oidcar
ti
lage,
cri
coi
det
c

30. Fr
ankst
ral
ingl
awi
ninot
ropesuse=mov
eup&l
eftcur
ve

31. Ri
ghtsi
dedoxy
gendi
ssoci
ati
oncur
ve=aci
dosi
slowph

32. Mostef
fect
ivemet
hodofcont
rol
li
nhduodenalph=br
unner
sgl
and-
bi
car
b

33. Fact
orr
egul
ateappet
it
e-sat
iet
y=gher
li
nfr
om epsi
loncel
l?

34. Massi
veasci
tescl
dpt–pl
eur
alef
fusi
on-
spi
romet
ry=

35. Copdonspi
romt
ery=

36. Her
oinusegi
vennal
oxone–abg=i
ncpco2r
espdepr
essi
on

37. Baggi
ngt
oremov
eni
tr
ogen=r
espv
olf
indi
ngt
lc/
frc/
fev
i/f
vc/
RV

38. Thor
aci
cout
letsy
sndr
ome=subcl
avi
anar
ty

39. Per
icar
dit
is=pher
nicner
ve
136

40. At
hel
eteuni
quef
indi
ng=i
ncsr
okev
olume

41. Tet
nusi
gGmosti
ndi
cat
ivef
act
or=par
kinj
urysoi
l

42. Woundi
nleggasf
ormi
ngor
gani
sm =cl
ost
ri
dium per
fri
nges

43. Antt
ri
angl
enecki
njur
y=pl
aty
sma0/
scm/
inf
rahy
oid

44. Weaknessf
lacci
dpar
aly
sisscenar
io=bot
uli
nism

45. Muscledidvi
deddur
ingt
hor
acot
omy=l
ati
ssi
mus
dor
si/
trapezi
us/s.
a

46. Thor
acot
omydonewhi
chner
vei
njur
e=t
hor
acodor
salner
ve,
axi
lar
y
et
c

47. Jawpai
nref
fer
edt
oear=ATN

48. I
nter
nalaudi
tor
ymeat
usner
vei
nj ed=7th+8th
ur

49. Damaget
omast
oidwhi
chner
vei
njur
edj
awr
efl ost=5thner
exl ve

50. Earl
obul
e=n.
s=gr
eat
eraur
icul
arner
ve

51. Or
bicl
uar
isoccul
i=n.
s=

52. Papi
ll
aryl
ightr
efl
ex

53. Safestbetablockerinafi
bwithasthmatic
pt
=atenalol
,car
vi.
prpranal
ol(car
diosel
ect
ive?)

54. Her
pessi
mpl
exencephal
it
isscenar
io=ctf
indi
ng=t
empor
all
obe

55. Expr
essi
vedy
sphasi
acase=br
ocas–f
ronall
obe

56. Opt
icchi
asm –bi
tempor
alhemi
anopi
a

57. Opt
ict
ract–homony
moushemi
anopi
a

58. Antspi
nalar
tysy
ndr
omecase

59. Scenar
ioofl
eftt
emor
al–r
ightnasalv
isi
onl
oss
137

60. Al
dest
ronedef
ici
ency=addsi
on=hy
pona+hy
perk

61. Connssy
ndr
omescenar
io=hy
per
nat
remi
a

62. Ri
ghtmi
ddl
elobeauscul
tat
ion=T4–T5

63. Gout
yptwi
thhear
tfai
lur
e=exacer
bat
ionofgout
=col
cichi
ne

64. Aor
ti
cregur
git
ati
onauscul
tat
ion=r
ight2i
cs

65. Youngptwi
thmeni
ngi
ti
ssi
gnswi
thr
ash=N.
meni
ngi
ti
s

66. Nsai
dwhi
chhav
elessgis,
e=i
bupr
ofen/
indomet
haci
n/di
clo

67. CaseofI
BSnsai
dsuse=b12absor
pti
ondec=b12def

68. Chol
era=i
ncchl
ori
desecr
eti
onchannel
sact
ivi
ty

69. I
ntest
inalf
lor
adi
stur
bance=cl
ost
ri
dium di
ff
ici
le

70. Ti
ghner
vesuppl
yant
-l
at=f
emor
alner
ve

71. Antcompar
tmentf
orear
minj
ury=muscul
ocut
aneousner
ve=c5-
c7

72. Antcompar
tementf
orear
minj
ury=pr
onat
ort
eresnot
heret
c

73. Gl
uteali
njgi
ven–f
ootdr
op=sci
eti
c/commenper
onealner
ve

74. Pneumoni
aol
dagei
nnur
singcar
ehome=or
gani
sm=

75. Hipprost
hesisdi
spl
acementl
igamenti
nvol
ve
=t
ransver
se/i
ll
iol
umber

76. Supr
acl
avi
cul
arf
ossader
mat
ome=C3

77. Si
ckl
ecel
l–ost
eomy
li
ti
s=sal
monel
la

78. Sal
monel
la=f
eaco-
oralr
out
e

79. Si
ckl
ecel
l-
anemi
aduet
o–r
bcsequest
rat
ion

80. Br
eastout
erquadr
entl
.n=antgr
ouporpect
oralgr
oup

81. I
faskedmedi
alquadr
ant=par
ast
ernalnodes
138

82. I
nst
ri
nsi
cmuscl
ehandsi
njur
ylowerbr
achi
alpl
exi
usi
njur
y=c8-
t1

83. Shoul
dercuf
frot
ator
s–i
ntr
otat
ionl
oss+=subscapul
ri
s

84. Lor
ezepam se=r
espdepr
essi
on

85. Mumpsi
sol
ati
onper
iod=5day
s(4-
4)

86. CaseofMumps+decpl
tbl
eedi
ngr
enalf
ail
ure=hsp/
imune
t
hroboct
opni
a

87. Febr
il
echi
ldhavefeverr
ashon9-
10monthv
accinatedall.
fol
lowi
ng
sy
smptomsduet owhich?=mmrvacci
ner
ubel
lasuspectd

88. Postspeenect
omy
=mcor
gani
sm pnemococcal
/h.
inf
luenza

89. Ecol
iinf
ect
ion=f
imbr
iaat
tachment
/hostdef
ense/
flager
ymov
ement

90. S.
ery
sipel
as=s.
pyogenes

91. Hear
tdefectnowI
nfect
iveendocar
dit
is=st
repv
iri
diansal
pha
heamol
yti
c

92. Femoralf
ract
urebl
oodl
ossdecbpt
henmai
ntai
nedbp
=bar
ror
eceptor
s

93. CBF=phy
siol
ogi
cdeci
cpduet
o=i
nit
iali
ncpco2=hy
per
vent
il
ati
on

94. sBP7opostt
ransf
usi
onbpmai
ntai
ned=t
prdec?

95. massi
vePul
monar
yembol
ism =echof
indi
ng=l
eftar
tri
um pr
essur
e
dec?

96. chemor
ecept
orl
ocat
ion=car
oti
dbody
/car
oti
dsi
nus

97. pancr
eas-
exocr
ine-
prot
iendi
gest
ion=t
rypsi
n

98. onl
yendocr
inepancr
eat
icsecr
eti
oni
s=gl
ucagonbyal
phacel
ls

99. f
aci
alner
ve=st
ylomast
oidf
oramen

100.ner
vet
oper
ior
bit
alar
ea=zy
gomat
ic/
inf
raor
bit
al/
buccal
139

101.f
ootdr
opsensat
ionl
oss=l
ater
alcompar
tmentofl
eg

102.sci
ti
cner
vedamage=ext
ensi
onofhi
p+f
lexi
onofknee

103.cushi
ngdi
seases=scenar
io=dmst
estdone=noef
fect

104.medi
alepi
condy
lei
njur
y=muscl
egr
oupi
nvol
vewr
istf
lexor
s/el
bow

105.dashboar
dinj
uryr
ta=pcli
njur
y

106.l
iv
eral
tenzy
meshi
gh=i
ncact
ivi
tyofcy
tosol

107.mi
tochondr
iaf
unct
ion=cel
lul
arr
espi
rat
ion&at
pfor
mat
ion

108.scenar
ioofptonl
mwhwi
thheamt
emesi
s=r
xpr
otami
nesul
phat
e

109.adenosi
neeffectonhear
tmoa=+chr
ontrpi
c/-
chr
ontropi
c/+dr
omot r
opi
c/i
nccor
nor
aybloodperfusi
on

110.i
prat
ropi
um br
omi
dei
nhal
er=r
ecept
ors=muscar
ini
c

111.poi
soi
ngcaseofant
ichi
ler
gict
oxi
drome=achr
ecept
orsbl
ockage

112.hear
tbl
ockcasesomeecgf
indi
ng=sl
owavconduct
ion

113.cutatv
olarwr
istabdpol
ices/
fpb/
o.p

114.t
apewor
minf
est
ati
on=r
x=ni
closami
deorpr
azi
quant
al

115.her
peszost
ergangl
ion=l
atgeni
cul
ate

116.di
lt
iazem s.
e=

117.femalehxofinnat
eimmunity=r
ecurr
entut
iinf
ect
ion=causesur
ine
fl
owdribbl
ingduetostr
uct
uralbybi
rt
hdefect…

118.hy
per
toni
csal
inemoa=i
ntr
acel
lul
ar-
int
rav
ascul
ar/
int
rav
ascul
ar-
I.
C

119.l
owani
ongap=hy
poal
bunemi
a

120.ani
ongaponl
yani
ons=cl
+hco3

121.mucoci
lar
ryescal
atorf
unct
ioncel
ls=ci
li
atedcol
umnarepi
thel
ium
140

122.f
ootdr
op=commenper
onealner
vei
njuy

123.smabl
oodsuppl
y=j
ejuni
um

124.ankl
eref
lex=S1S2

125.hy
ocal
emi
a=v
itddef

126.pt
hdefscenar
io(
deccapt
hdeci
ncph)

127.ni
ppl
eder
mat
ome=T4

128.ppImoa=h/
kat
pasepump

129.r
xofdka=i
mpor
tantEl
ect
rol
yter
epl
acement
=pot
assi
um r
ep

130.t
bcasef
rom i
ndi
a=dxsput
um af
bsmear

131.chi
ldwit
hgast
ri
closs,
vomit
ing,
met
abol
ical
kal
osi
sel
ect
rol
yte
di
stur
bance=duetol
ossofhcl

132.ccbaml
odi
pinese=pedaledema

133.r
ashaf
tert
aki
ngmedi
ntonsi
li
tt
is=amoxi
cil
li
n–pmpduet
o(EBV)

134.toxi
cmegacol
on–dxhi
erschpr
ungdi
seases=my
ent
ri
corauer
bacs
pl
exus

135.ar
trydamagei
nrect
ussheat
h=i
nfepi
gast
icv
essel
-ar
ty

136.moaorNEi
nsept
icshock=v
.c

137.heamost
atsi
sinj
urymostabudandcel
latend=f
ibr
obl
ast/
plat
elet
pl
ug/N/L

138.chr
oni
cgastr
it
isonbiopsypr
edomi nantcel
l
=l
ymphocyt
es/
macrophages/mult
inucleat
edgiantcel
l/
neat
rophi
ll
s

139.gl
ucagonbywhi
chcel
l=al
phacel
lofendocr
inepor
ti
on

140.abdoimi
nalt
rauma–herni
ati
onofintest
inei
ntochestt
hrough=t
7-
t9l
evel
/domesoddiaphr
agm/cent
ralt
endon
141

141.scenar
ioofdxofpheochr
ocy
toma–al
phabl
ockade

142.my st
hani
agravi
ssuspect
edcaseopt
icneut
it
isf
indi
ngs=duet
o
lossofmyel
anati
on

143.woopi
ngcough=nasophar
ngealswab

144.acr
omegal
ycasescenar
io

Note;addev bm 5questi
on,restofotherquest
ioni
fuhaveorcompl ete
themorestems&opt ionsifuhav e,someofquesti
onarepoorrecal
lstry
toaddinthese..ki
ndlyconfi
rmt hekeyalsothi
sisj
ustancoll
ecti
vef or
m
notaconfir
medone…bestofl ucktoall
…mayALLAHsucceedal lofusin
ourgoal
s…. staybl
ess
142

DEC 2019
1-Abduction of the arm > supraspinatus
2-axillary N injury > Teres minor
3-SMA supplies > ascending colon
4- Direct hernia > medial to inferior epigatric artery
5-long saphenous vein > anterior to medial malleolus
6-Froment sign > adductor pollicis
7- Loop diuretic > Na/k/2Cl
8-Loop diuretic > thick ascending loop of henle
9-Thiazide > early part of distal convoluted tubule
10-stab in antecubital fossa > median N
11-fracture in head of fibula > common fibular nerve
12- absorption in distal ilium > B12
13- Pulmonary congestion > J receptor
14- patient presents with chronic cough, which receptor can cause this > irritant
receptors
15- patient with HR 40-60 bpm , which part of conducting system can cause that > A-
V node
16- STEMI with Bradycardia > Rt coronary artery occlusion
17- mean arterial blood pressure formula ( MAP) > diastole + 1/3 pulse pressure
18- Henry 's law
19- mechanism of compensation of BP in supine hypotension, increase of what ?
preload , contractility, afterload , cannot remember the other choices.
20- First response in hypovolaemic shock > baroreceptors
21- drug cause hypokalemia > Frusemide
22-Patient who was on antacid , which of the following drugs can be affected >
Didoxin
23- Lorazepam mechanism of action > potentiates GABA
24- insertion of peroneus longus > 1st metatarsal bone
25- which one is part of thenar muscle > abductor pollicis brevis
26- Patient -IV drug user-fever, new systolic murmur ,causative organisms > staph
aureus,
27- bloody diarrhea with ascending weakness > campylobacter jejuni
28- Auscultation area of aortic valve which intercostal space > Second Right ICS
29- what of these can be compressed by Lt atrium enlargement > esophagus
30- structure presents in T4/T5 level > can't remember the choices
31- sensory level of xiphisternum >T6
32- sensory level of umbilicus > T10
33- wound in posterolateral part of back > latissimus dorsi
34- Common site of ectopic pregnancy > ampulla
35- Thoracic duct passes through diaphragm with which structure > aorta
36- cause of peptic ulcer > increase HCL secretion , increase of gastrin H, decrease
Bicarbonate
37- Causes of low anion gap acidosis > hypoalbuminaemia
143

38- Patient with sickle cell diagnosed with osteomyelitis – causative organism >
salmonella
39- when the child can go back to school Rubella > 5-7 days after appearance of
Rash
40- virulence of staph > protein A
41- incubation period of varicella > from 10 to 21 days
42- antidote of Heparin > protamine sulphate
43- reversal of warfarin > dried prothrombin complex > no choice of vit K.
44- mechanism of action of lidocaine > inactivation of Na channels.
45- mechanism of action of Na nitropruside > release NO.
46- mechanism of action of PPI > inhibition of H/K atpase
47- Common organisms in nursing home >legionella
48- neonate with unilateral eye discharge became purulent > Chlamydia
49- hormone is secreted by post pituitarty > vasopressin
50- hormone works in dehydration > antidiuretic H
51- case of hyponatremia and hypokalemia , defect in > mineralocorticoid.
52- abnormality of which hormones can cause hypertension with other mentioned
symptoms of cushing syndrome > corticosteroid
53- cause of cushing syndrome > pituitary adenoma
54- case of vertigo, ipsilateral hemiataxia, dysarthria, ptosis and miosis > occlusion of
posterior inferior cerebellar artery (PICA).
55- contralateral homonymous hemianopia > optic tract
56- lesion in temporal lobe > superior quadrantanopia
57- anti psychotic drugs acts on which receptors > Dopamine
58- hypocalcemia causes increase of which hormone > parathyroid H
59- main cell in acute inflammation > neutrophil
60- macrophage is derived from which cell > monocytes
61- nerve injury cause diplopia and eye deviation downward and outward >
oculomotor nerve palsy
62- Weakness in upper limbs more than lower limbs > central cord $
63- A woman on OCP became pregnant after she started new antiepileptics
medications > carbamazepine
64- Influenza enter the cells through > haemagglutinin antigen.
65- which cell is responsible for gas exchange > pneumocyte type 1
66- which cell is responsible for surfactant secretion > pneumocyte type 2
67- Facial nerve injury but frontalis muscle is spared , nerve supplying orbicularis
oculi > Zygomatic , other answers were Temporal, buccal, Mental and Cervical.
68- case of diarrhea with history of splenectomy , which these capsulated organisms
can cause this > E coli, other answers were H influenza, strept pneumonia, ….
69- Anions to calculate anion gap > Chloride and bicarbonates
70- In which level IVC passes through the diaphragm ? T8
71- In which level esophagus passes through the diaphragm? T10
72- Drug causes rupture tendon > ciprofloxacin
73- weakness of flexion of thumb, index and middle finger > ant interosseus nerve
injury
74- septic arthritis > staph
75- dental extraction and rheumatic H > strep pyogenes
144

76- mechanism of action of propofol > GABA stimulation


77- Q about malaria > Plasmodium falciparum
78- foot drop > common peroneal N injury
79- Direct inguinal hernia ,weakness in > transversalis facia
79- first drug you start with in thyroid crisis > propranolol
80- vomiting > metabolic alkalosis
81- movement of molecules in dialysis > simple diffusion , ultrafiltration , ….
82- Thoracic outlet $ > compression of subclavian artery
83- Drug causes hemolytic anemia > mefenamic acid
84- Drug causes cholestatic jaundice > Co-amoxiclav
85- Function of mitochondria > Energy production
86- mechanism of hypersensitivity type 3 > IgG, IgM , immune complex attacking
basement membrane, IgE
87- pain in Jaw radiated to ear > auriculotemporal N
88- structure presents in first layer of planter side of foot > can't remember the
choices
89- taste sensation of ant 2/3 of tongue > facial N
90- acute gout with HF > colchicine
91- acute gout with NSAI with the least GI side effect > Indomethacin , Naproxen ,
diclofenac
92- mechanism of cough in ACEI > bradykinin
93- blood supply of meckel's diverticulum >SMA
94- Muscle helps in forced expiration > rectus abdominis
95- case of child of acute lymphoblastic leukemia
96- sella turcica presents in > sphenoid bone
97- In Lumbar Puncture , next structure after piercing skin and subcutaneous tissue >
supraspinous ligament
98- vasoconstriction is due to stimulation of > alpha1 , alpha2, B1, B2
99- Frank starling law > I can't remember, but I think HF > decrease contractility
100- Infection After circumcision > deep inguinal LN , superficial inguinal LN
101- 1st line of treatment in asthmatic attack > sulbutamol
102- Ipratropium bromide mechanism of action > muscarinic receptor
103- Giardia mechanism of attachment > suckling
104- mechanism of hypertonic saline > water comes out of the cell
105- hip injury causes weakness in hip flex, abduct, and laterally rotate the thigh (as I
remember) > sartorious
106- how to prevent spread of Cl. Difficile to others > wash hand with soap ,
treatment with metronidazole
107- site of action of ADH > distal convoluted tubule and collecting duct
108- site of action of PTH > distal convoluted tubule
109- case of meningitis in adult > negative diplococcus (N. meningitis)
110- cell important in clearance of particles > ciliated columner
111- when Aortic valve close > isovolumetric relaxation
112- hilar lymph nodes drain in > bronchopulmonary LN
113- Nerve supply above eye brow till the vertex of the scalp > supraorbital N
114-when light is on lf eye , the lf pupil dilates and Rt eye constrict ,,,,
When the light is on Rt eye, both eyes constrict >>> optic N injury
145

115- most common aneurysm is >> anterior communicating artery


116- Flexor tendon sheath continues with >> The little finger
117- small pulmonary vessels damage with ischemia >> increase in physiological
dead space
118- Fetal hemoglobin shifts oxygen saturation curve to left because > increase
affinity to oxygen
119- Chemo receptors presents in > aortic body
120- lip swelling with no improving on adrenaline, which drug is implicated mainly as
cause > BB
146

FRCEM PRIMARY JUNE `19 RECALL


DR. MOHD SAMIULLAH
DR. QAMAR AKTHAR

1. SMA thrombosis – Ascending colon


2. Pneumoperitoneum Mc d/t – Duodenum
3. FAST Scan – Sub hepatic space
4. Injury to pituitary gland causes damage to – Facial nerve
and maxillary artery
5. Sella turcica – sphenoid bone
6. Foramen spinosum – meningeal branch of maxillary artery
7. Hyperextension injury – central cord syndrome
8. Antecubital fossa – medial nerve injury
9. Wrist drop – radial
10. Medial epicondyle injury
11. Lateral epicondyle injury
12. Froment sign – adductor pollicis
13. Opponens pollicis – median nerve
14. Superficial structure to flexor retinaculum – ulnar
nerve
15. Aorta opening -T12
16. Oesophagus opening – T8
17. Xiphisternum injury, sensory loss – T8
18. Internal capsule stroke – MCA
19. Anterolateral thoracotomy – Internal mammary
artery
20. Taste from anterior 2/3rd of tongue – chorda tympani
21. Uvula deviation – vagus nerve injury
147

22. Auriculotemporal nerve


23. VZV reactivation, facial palsy – geniculate ganglion
24. Valgus injury to knee – medial collateral ligament
25. Sciatic nerve sensory check – dorsum of foot
26. First layer of sole, injury to – medial plantar artery
27. Greater trochanter – gluteus Medius attachment
28. Lateral rotation of hip – psoas major
29. Adduction of hip with sensory loss in medial thigh d/t-
obturator nerve
30. During LP structure pierced after subcutaneous fat –
Supraspinous ligament
31. Fibular head # - Common peroneal nerve
32. Direct inguinal hernia – weak posteriorly d/t –
transversalis fascia
33. Direct inguinal hernia – medial to inferior epigastric
artery
34. Penis drains to – deep inguinal LN
35. L1 # - Internal oblique and transverse abdominis
nerve supply effected
36. Thoracic outlet syndrome – subclavian artery
37. Horner’s syndrome artery involved – vertebral artery
38. Diplopia on downward gaze – III N palsy
39. Adducted eye – VI N palsy
40. Eye deviated out and down – III N palsy
41. Expressive aphasia – frontal lobe stroke
42. Pontine bleed -? motor deficit? reduced level of
consciousness
43. Orbit infection, posterior spread prevented by -?
orbicularis oris?
148

44. Berry aneurysm MC – anterior communicating artery


45. Temporal lobe lesion – superior quadrantinopia
46. L homonymous hemianopia – R optic tract lesion
47. Peroneal retinaculum – calcaneum attachment
48. Prominent cervical spinous process- C7
49. GTCS with shoulder joint dislocation
50. Ulnar nerve –?? C7/T1, T1/T1 injury
51. Swallowing phase of oesophagus -? vagus
52. Distal end radius #, tendon injury
53. Pt with HTN, headache, sweating – which visual field
defect
54. Gag reflex absent – CN IX
55. Hoarseness of voice – RLN
56. Knife injury to forearm –? ECRL
57. Ampulla of vater – 2nd part of duodenum
58. Systemic vascular resistance is d/t – Arterioles
59. Blood brain barrier – endothelial cells
60. Osmotic pressure is decreased by – albumin
61. S2 and ECG correlation – T wave
62. S1 is best heard – L- 5th ICS
63. Decreased gastric emptying – low duodenal Ph
64. Endocrine substance of pancreas – glucagon
65. Secretion of which hormone is not affected in
pancreatitis – CCK
66. B12 absorption – pernicious anaemia
67. Pernicious anaemia is d/t – parietal cell
68. Dicrotic notch is d/t – closure of aortic valve
69. RTA with hypotension – Baroreceptors
149

70. Pt. with metabolic acidosis, RR increases d/t: Medulla


oblangata
71. Vasovagal – Ach
72. Inotropes effect on Frank Staling curve: shifts
upwards and to left
73. Frank starling law – Preload & force of contraction
74. STEMI with bradycardia – RCA
75. Diazepam overdose, pt. with bradhyaponea, antidote
given, RR is increased, what decreases – PCO2
76. Emphysema – increase in physiological dead space
77. Spo2 100% with low pao2 – CO poisoning
78. Valsalva – Vital capacity
79. Addison’s disease – Decrease in mineralocorticoids
80. Pt. with heart failure and pulmonary oedema- J
receptors
81. Multiple drugs -? heroine
82. PTH effect in hyperphosphatemia – decreases PO4
reabsorption in PCT
83. Furosemide – inhibit Na Cl reabsorption in Thick
ascending loop
84. Furosemide – acts on Na/K/2Cl receptors
85. Proton pump – primary active transport
86. Low anion gap metabolic acidosis – Hypalbuminaemia
87. Pt in sepsis with ABG showing HAGMA, corrected
with NaCl, becomes NAGMA d/t: Cl excess / lactate
reversal
88. Insulin – increases K+ reuptake by cells
89. CaCl2 in Hyperkalaemia – Stabilizes cardiac
membrane
150

90. Why Creatinine used to estimate GFR – Steady state


pl. concentration / filtered freely in glomerulus
91. Reflux esophagitis with epigastric pain is d/t –
decreased HCO3
92. Galactorrhoea – prolactin
93. Pt with hypersalivation due to sympathetic effect,
what is seen in saliva – increase in Cl / mucous
94. Diabetes insipidus – decrease in pl. concentration
95. Pt. in hypotension – Increase in ADH secretion from
posterior pituitary
96. In optic neuritis decreases conduction is d/t –
demyelination
97. Axonal speed is increased by – myelination
98. Pt with hypocalcaemia – increase in PTH
99. Hypertonic saline MOA – water moves out of the cell
100. B/L ankle oedema, what extracellular cation increases
in cells – Na
101. Glucose absorption – PCT
102. APLS resuscitation scenario, ODC left shift in infants is
d/t low P50
103. Chronic Smoker – Decreased mucocilicary clearance
104. Pt in tachycardia, what increases ventricular filling –
AV nodal delay
105. Av node opens in – Isovolumic relaxation
106. Abundant cells after haemostasis – Platelets
107. Vasoconstriction – serotonin
108. Diffusion of urea in dialysis – ultrafiltration
109. Accessory muscle of expiration – rectus abdominis
110. Contraction of GB – CCK
151

111. Hypocalcaemia – Albumin is used to calculate


corrected value
112. SIADH cause – Fluoxetine
113. Septic shock pt. – alpha agonist inotrope is used
114. Hyperkalaemia & AKI causing drug – Candesartan
115. Ac. Gout with heart failure DOC – Colchicine
116. Ketamine MOA – NMDA
117. Antihistamines causes sedation d/t – crossing BBB
118. Diazepam MOA – GABA agonist
119. Vit B12 deficiency – Nitrous oxide
120. Lithium toxicity
121. Jaw pain with migraine with extrapyramidal
symptoms – Metoclopramide
122. Diltiazem effect – negatively ionotropic
123. Risperidone – D2 receptors
124. Achilles tendon rupture – Ciprofloxacin
125. CDC causing antibiotic – Ciprofloxacin
126. Maculopapular rash causing antibiotic – Amoxicillin
127. Vertebral # is d/t – Omeprazole
128. Ac. Gout scenario, which drug can be given with least
GI side effects – Naproxen / ibuprofen
129. Hypocalcaemia causing drug – Glibenclamide
130. Hyoscine contraindication – Tachycardia
131. Thyrotoxicosis with AF, DOC – Propranolol
132. Child ate berry, presented with blurring of vision, dry
and hot skin
133. Ipratropium bromide MOA – Muscarinic
134. Antihistamines case sedation d/t: Cross BBB
135. DVLA driving licence –? DM pt. on OHG
152

136. ACE inhibitors causes cough d/t – Bradykinin


137. Postural hypotension d/t – Thiazides
138. Asthma worsened when started new
antihypertensive, d/t
139. H1N1 entry into cells – Haemagglutinin
140. Pregnant lady with pl. vivax, DOC – Mefloquine
141. Meningitis in old age, extra broad-spectrum
antibiotics is added for – Listeria
142. Malaria – Giemsa stain
143. Malaria causing drowsiness – pl Falciparum
144. GBS – Campylobacter jejuni
145. CDC prevention – washings hands with soap and
water
146. Cat bite infection, gm negative organism
147. 2year with pertussis – Throat swab
148. Measles infectivity period – 4days after the onset of
rash
149. 9months with full vaccinations till date, develops rash
d/t – Rubella
150. HIV patient – BCG is contraindicated
151. Erysipelas MC caused by- Streptococcus pyogenes
152. Meningitis – Gm negative diplococci
153. Staph. aureus – Protein A
154. Giardia – Suckling disc
155. MC cause of osteomyelitis – Staph. Aureus
156. Osteomyelitis cause in Sickle cell child - Salmonella
157. IVDU with Infective endocarditis cause – Staph
aureus
153

158. Infectious mononucleosis predominant cells –


Lymphocytes
159. Old pt. with DM presents with cough and sputum, MC
cause – Streptococcus pneumonea
160. 9year old with recurrent chest infections with
premature cells – ALL
161. Complement deficiency increases susceptibility to –
Streptococcus infections
162. Nut allergy – IgE
163. Predominant cell in anaphylaxis - mast cells
164. Neisseria meningitis – IgA protease
165. Wound healing maturation phase – fibroplasia
166. Vomiting child with alkalosis with hypokalaemia d/t -?
Hcl loss
167. Compensation for chronic respiratory acidosis – HCO3
increases
168. Mitochondria – ATP production
169. Gastrin hormone
170. Asthma with loose motions & vomiting since4 days,
what increases???
171. Power is increased by – increasing in sampling size
172. Type 1 error
173. New drug better than ramipril, mortality unknown,
what to do.? RCT /prospective
174. Study group vary difference
175. Control subject in experimental group
154

Recall FRCEM Primary June-2019

compiled by Dr. Bushra Sabahat Khan


1. What movement at wrist joint you perform to access Median nr injury?

Entension , Flexion, abd of fingers, add of fingers....

2. Blunt abdominal trauma, FAST done, where does the fluid accumulates?

Subphrenic recess, Hepato renal recess, Pouch of Douglas.....

3. Vertebral level of opening of aorta in diaphragm?

T12

4. Vertebral level of opening of Esophagus in diaphragm?

T10

5. Dermatome at xiphisternum?

6. Pt. came with uvula deviation and difficulty in swallowing, which nerve is affected?

CN-lX, CN-Xll, Phrenic Nr.......

7. DOC for P. Vivax?

8. Illness from cat scratch ,what will be the causative gm -ve bacteria?

9. Which stain is used for malarial paracite in Microscope?

Geimsa stain

10. Median Nr. suppliese to which of the following Muscle?

FRC, Opponens Pollicis....

11. Tendon ruptured, what is the causative A/B?

Ciprofloxacin
155

12.Dx test for Pertusis?

Nasal swab, Parnasal swab, throat swab.....

13. Woman with Galactoria and Amenorrhea,cause?

Prolactinoma

14. ↓Ca++, which hormone will response?

↓calcitonin, ↑Calcitonin, ↑PTH

15. On which receptors Lorazipam act on?


16. MOA of Ketamine?

17. ↓Na, ↑K... which hormone is involved?

18. Bitemporal Hemianopia , where is the lesion?


Optic chiasm

19. Cause of Inf. Homonymous Quadrianopia ?

Damage of Optic rasiation in Parietal cortex

20. Nr. supply to ant. thigh?

21. Hip injury Sciatic Nr. damaged, which nr. in the the foot is to access for damaged?

Sural, Femoral , Tibial, Sephanous

22. Foot drop, which nr. is damaged?

23. Which stucture is damaged superior to F. Retinaculum?

24. Tree surgron , pain in medial epicondyle, which group of muscle are affected?

flexors of wrist , extensors of wrist

25. Inguinal hernia is d/t weakness of which ligament?

26. Direct inguinal hernia? medial to inf. epigastric vessels?

27. Drug abuser came with murmur, which organism ?


156

28. Area of auscultation for mitral valve?

29. Pt. after diarrohea developed weakness of lower limb and trunk, which organism?

30. Child with vomiting, Rx for acidosis, reason for acidosis?

↓K, HCl loss

31. Thoracic outlet syndrom , vessel affected?


32. Drug that develops SIADH ?Fluexitine
33. Pt. came with thyroid crisis .Rx at A&E?
Propanolol, Carbimezapin
34. MOA of Diltiazem?
35. Pt. came with hoarsness of voice ,profused sweating?
GH adenoma/ Pituitary adenoma
36. Urea filteredin kidney by?
Osmosis, Simple diffusion
37. Which NSAID causes anemia (hemolytic)?
38. what mechanism does NSAID cause GERD?
39. Mechanism of secretion of HCl?
40. Function of Mitochondria?
unrolling of new protein, RNA-DNA Production
41. Asthma pt. with diarrhoea and vomiting for 4 days, which hormone will inv
olve to deal with the symptoms?
42. Pain in ear and jaw, which nr. is involved?
auriculotemporal, inf.alveolar, sup. alveolar
43. Which organism causes Erysipelas?
grp. A streptococcus
44. Which vessel is damaged in Em thoracotomy?
45. Anaphylaxis ,type of HS?
46. HS type-1, which Ig ?
Ig E
47. Maturation stage of wound healing?
157

48. Hemostasis is acheived, now which cells will stay longer?


PMNL, Monocytes, Macrophages
49. Expressive dysphasia , which lobe is involved?
50. SE of hydrochlorothyazide?
51. What pathology is there in MS?

↓ conduction in unmylinated Nr., unmylinated axons excites for shorter durati


on
52. Injury of superficial layer of planter side of foot,which muscle is involved in
injury?
53. Pt.with steatorrhoea , which exocrine pancreatic enzyme is not involved?
54. Child with soar throat, treated with A/B , developed rash, which A/B?
Cipro, Cephalo, Erythromycin
55. Froment sign, Which Nr. is involved?
56. Sensory supply of ant. 2/3rd of tongue?

57. Factor involved in ↓ Gastric emptying?

58. Rx of acute gout in pt. with HTN & DM?


Cholchicin
59. Cough developed after using ACEI,phenomenone involved?

60. Pt. Dx with ↑K+ , Ca-Gluconate is adviced, reason of using Ca-Gluconate?

61. Why insuline is given in DKA?

To ↓K+, ↓ acidosis, ↓ Glucose

62. Which cytokine is involved in Rh Arthritis?


TNF@ , IL-1, IL-10
63. Site of Vit.B12 absorption in GI?
64. Retroperitoneal structure damaged, what is the structure?
Duodenum
65. Reason of sedation with anti histamine?
crosses BBB
66. Which structure prevent crossing BBB?
158

Ependydimal cell, Choroid Plexus


67. What muscle is used in forced expiration > forced inspiration?
int. Oblique, Rectus abd. , Transverse abd.
68. Which aneathetic causes aneamia?
NO, Kitamine
69. Splenetomy done, pt. is prone to which organism?
70. Blood supply of ascending colon?
71. What phase of cardiac cycle is at Mitral valve closure?
72. Which NSAID has least SE on GI?
73. Pt. with pul. edoema has tachypnoea, Reason?
J receptor stimulation

74. Child with bleeding-bruising S/S ,CBC shows ↓Hb with immature cells on
peripheral film, Dx??
Heamophilia, Lymphoblastic leukemia
75. Question regarding Heqmophilia?
76. Where from ADH is secreted?
ant. pituitary , post pituitary
77. Sella turcica is located in which bone?
78. In LP which structure needle will pass through after skin and superficial fas
cia?
79. 9months old baby came with symptoms, which vaccine is not given by yet?
80. Pt. came with dry mouth, uninary retention, tachycardia, what receptors are
excited?

a1 or ß1

81. Frank sterling law?


82. Infecton after circumscision, which LN drains the area?

83. AIDS pt. with ↓CD4 count, which vaccine is contraindicated?

BCG

84. Site of action of Loop’s diuretics?

85. Virulent factor of H. Influenza?


159

86. Giadria attached with which structure?


Flagella, Suckers
87. Ipratropium bromide acts on what receptors?
Muscarinic
88. Mechanism of Hypertonic saline act on cell?
89. C/I of Hyoscine?
90. Pt. came with septic shock, Rx given, pulse oxymeter shows normal PaO2
& BP. ABGs shows ↓PaO2 and acidosis. Reason?

91. Cavernous sinus thrombosis, which Nr. is affected?


92. Damage to Parotid gland, which of the vessel will be affected?
Facisl Nr.& Maxillary A
93. Sick pt. came fro recent tour of Nigeria / Somalia, fully vaccinated, what is t
he causative organism?
P. Falciparum
94. Sick looking pt. with chest pain, Bradycardia, ECG shows changes, what ar
tery is occluded?
RCA
95. Quesiton related to Sample Size?
96. Question realted to Cohot Study?
97. Nr. passes from foramen Spinosum?
98. Upper arm injury, which muscle is damaged?
Brachioradialis
99. Spiral groove injury of Humerus, which Nr. is damaged?
Radial Nr.
100. Valgus injury of knee , which ligament is damaged?
101. Wound at anticubital fossa, which Nr. is damaged/
102. Eye got damaged,fixed to medial and lower side, which nr. is affected?
103. Question related to diplopia?
104. Hip injury......... Muscle damaged?
Ilioinguinal, Sartorious
105. Question related to Consensual Light reflex?
160

106. Question relsted to DVT?

107. Pt. with COPD, which value is ↑ed in Spirometry?

108. Drug thst causes Clostridium Difficile Colitis?


109. Mechanism of prevention from Chlostridium?
Hand wash with soap, sterilization
110. Osteoprosis + vertebral involvement , Drug?
111. Site of action of ADH in nephrone?
112. Site of action of PTH in nephrone?
113. Question related to Lesion in Pons?
114. Pt. with H/O fall, Hyperextesion of neck injury, upper limb weakness > low
er limb weakness?
central cord synd.
115. Diabetic pt. with small wound in leg area, causative organism?
116. In eyelid which structure prevents the spread of infection?
Tarsal plate

117. Question related to sypmtoms of SIADH, pt. after RTA came with ↓Na+ an
d S/S?
118. Which muscle isinserted at greater trochentor of femur?

119. Conditions having ↓ anionic gap?

120. Ampulla of veter tumor, at which part of duodenum does it open?


121. HS Type 2?

122. Pt. came with ↓R/R, and in shock, Rx provided & pt. revived. What vslue i
n ABGs will be raised after revival?
123. Hpotension, 1st receptors to be excited will be?
Baroreceptors, Aortic bodies, Carotid bodies, chemoreceptor, J receptor
124. Biochemical values in D.Insipidis?

↑s Osmolality, ↑s Na+, ↓u Na+

125. Rx of N. Meningitidis virulence factor?


126. Effect of H1- blockers on HCl secretion?
161

127. Suitable ß- blocker for asthmatic?

128. Point of auscultation of Aortic valve?


129. Valgus stress .... Ligament?
130. Nr. damaged in arm, loss of Finger flexion, What is the nr.?
131. O.Myelitis in young child , which is the organism?
132. Valsalva meneuvar, what affact does it cause on Lung vol.?

133. Pt. with HF, SOB, oedema,what ICF ions is ↑ed?

134. DOC for Rx of Angioedema?


135. Blood supply of SA node?
136. Virulence factor that cause damage in Rh Arthritis?
RA factor, IL-1
137. Which respiratory cells help in clearnce of Dust & Dirt particle?

138. Cause of ↑ed mucus production in smokers?

139. Pt. is unsble to flex the knee,which nr. is damaged?


140. Vaccine that is CI in pregnancy?
141. Question of Newborn , scenario relted with O2 dissociation respiratory cu
rve, choices included,
Carboxy Hb, @-chain of Hb

142. RTA with S/S ,↓Na+, ↑K+, ↓H2O,what is the cause?

↑ADH, ↓ADH, oxytocin

143. Question related to the mechanism of Co-transport?


144. Question related to Standard deviation?
145. Swallowing reflex, Esophageal phase, which nr. is involved?
146. What changes comes in composition of saliva, if we chew the food for lon
g?

↓K+, ↓Na+, ↑K+, ↑Na+

147. What phase of ECGs is at the closure of Aortic valve?


148. To slow down Ventrical filling, at what level nr. conduction should be slow
ed down?
162

SA node, AV node, Purkinji Frb., Bundle branch,


149. Question related to Muscle of Thenar eminence?
150. Nr. supply of the lateral of the foot?sural Nr,???
FRCEM Primary December 5 2018 Recall 163
This is all I could remember and highlighted answers are the one I chose, I am not sure those are
the right answer.

1) Fall on the shoulder and not able to do the drop test- Supraspinatus, infraspinatus, teres minor ,
subscapularis,deltoid
2) Loss of sensation around deltoid area- nerve injury. Which of the muscle weakness?
Teres minor, infraspiantus, supraspinatus, subscapularis
3) Fall injury and loss of abduction and flexion of arm – C4C5 , C5C6 , C7, C8T1
4) Anterior dislocation of Hip Joint- What is the most stabilizer of hip joint?
5) Head injury , structure pass through Foramen Spinosum- Meningeal branch of V3, Maxillary ,
Trochlear, Opthalmic branch of trigerminal
6) Patient on Furosemide – the site of drug action on renal tubules? PCT, DCT, Loop of Henle, Thick
ascending loop, Thin ascending loop
7) Patient with Hypocalcaemia – Increased PTH- act on which site of renal tubules? Not remember
the stems
8) Patient was Given GTN spray in angina, mechanism of drug- increase cGMP
9) Patient was given aspirin in MI- mechanism of action on platelet- increase prostacyclin, inhinbit
cycloxygenese, increase thrombosane
10) During blood transufusion, patient develop back pain – etc. sign of acute haemolytic anaemia-
Why? Delayed typed hyptersensitivity, T-cell mediated, Complement , systemic Immune
complex, IgM mediated
11) Patient with Spigelion hernion due to muscle weakness of- transversus abdominus aponeurosis
12) Patient stepped on glass at heel and need ankle block- saphenous, sural, deep peroneal, superficial
peroneal
13) Stab wound at antecubital fossa and injury of brachial artery, complete transection of which
artery? Profundi branchii, radial, ulnar collateral,
14) Same scenario- which collateral will be spared? Profundi brachii, palmar branch, dorsal arch,
radial
15) Stab wound to the abdomen and injury to supracolic peritoneum- stomach, jejunum, colon, first
part of duodenum
16) ARDS in new born- which law applied? Laplace law
17) Premature new born was given the drug ? monosaccride, disaccride, polysaccride, phospholipid
18) Patient with Crohn’s disease with IDA- distal ileum, proximal ileum, duodenum
19) Shoulder joint ligament injury and cannot elevate the shoulder- acromioclavicular,
coracoclavicular, glenohumural, coracohumeral
20) Blood supply of pancreas
21) Tachypnoea in Pulmonary embolism – 2 questions – first answer C-fibre receptors and another
answer J receptors in second questions
22) Heart failure patient- Frank-starling law- downward shift, upwardward shift, no shift, left , right
shift
23) Medical student asked you how to calculate Cardiac ouput. HR-60 – SV
24) Heart failure patient ask about preload- End systolic volume of atria, end systolic volume of
ventricles, end diastolic volume of atria, end diastolic volume of ventricles
25) Physiological response in haemorrhagic shock- don’t remember the stems
26) First response in hypovolaemic shock – baroreceptors
27) Patient who was on antacid . Which of the following drugs can be affected? I chose Digoxin-don’t
remember other stems
28) Which of the drugs contraindicated in bowel obstruction? Cyclizine, promethazine,
metocloperamide, ondansetron
29) Mechanisms of action of following antiarrhythmics in 4 questions
30) Atropine – M3 receptors
31) Amiodarone- prolong Action potential
164 32) Adenosine- Delayed AV conduction
33) Flecanide- Sodium channel blocker
34) Adrenaline dose in cardiac arrest- 1:10,000
35) Patient with advanced liver failure- which of the following drugs does not need dose adjustments?
Furosemide, morphine, diazepam, digoxin, bisoprolol
36) Patient with infectious tenosynovitis pain on flexion of wrist and metacarpal phlengeal joints. Site
of infections – carpel tunnel, Gryon’s canal, thenar eminence, hypothenar eminence
37) Injury to lateral epicondyle and MRI show injury to the muscle which attached to base of 3rd
metacarpal- extensor carpi radialis brevis
38) Lorazepam was given in Status epilepticus – mechanism of action – binds GABA receptors,
potentiates through GABA, potentiates through glutamate
39) Child presented with multiorgan failure and TSS. History of tonsillitis a few days ago. Causal
organisms- Strep Pyogenes, Staph aureus, others ???
40) Patient -IV drug user-fever, new systolic murmur- causal organisms- staph aureus, strep virridans
41) The patient presented with vomiting and diarrhea 1 hour after eating food. Staph aureus
42) Patient present with hoarseness and uvular deviation on phonation- recurrent laryngeal nerve,
vagus, phrenic ,ETc
43) Anatomy location of great saphenous nerve- anterior to medial malleolus
44) Pain on base of little finger – Lunate, Hamate, Sphenoid, Trapezium, Trapezoid
45) Auscultation area of aortic valve which intercostal space- fourth Right, Fourth Left, apex, Second
Left, Second Right
46) Depolarization of phase zero action potential- which ion movement – rapid calcium, slow sodium,
rapid sodium, rapid potassium etc.
47) Normal physiological split of second heart sound in deep inspiration- delayed aortic valve closure,
delayed pulmonary valve closure , etc
48) 3 heart sounds- ventricular ejection, rapid atrial filling, isometric ventricular contraction,
isometric ventricular relaxation , rapid ventricular filling
49) Cause of oedema in Pulmonary Oedema – decrease artrial pressure, decrease capillary pressure,
increase central venous pressure, lymphatic obstruction
50) RTA – x ray show raised left hemi diaphragm- phrenic
51) Thoracic duct injury- Which opening does Thoracic duct penetrate through the diaphragm?
Aortic, Venal, oesophageal, right dome of diaphragm
52) IVC penetrate the diaphragm at which level – T8
53) Aorta penetrate the diaphragm at which Level- T12
54) Chocking of a child at T4/5 why- Arch of aorta, right bronchus,etc
55) Left atrial enlargement compresses on - oesophagus
56) Patient with humurus fracture -multiple myeloma – Increased IgA, IgD, IgE, IgG, IgM
57) Contents of rectus sheath- pyrimidalis
58) Stab woud at 5th intercostal space mid axillary line- Lattisimus dorsi, serratus anterior, P major, P
minor
59) Bleeding in ectopic pregnancy- internal iliac, external iliac, coeliac
I chose internal iliac which gives branch to uterine artery
60) Common site of ectopic pregnancy – ampulla, fimbriae, ovary, isthmus
61) Patient came with oeophageal varices from left gastric vein- collateral – azyogs system, portal vein,
hepatic vein
62) Common causal organism of acute cholecystitis – E-coli
63) Hormones causing gall bladder contraction – CCk, glucagon, gastrin etc.
64) Hormones causing PU – Gastrin
65) Patient with sickle cell diagnosed with osteomyelitis – causal organism – salmonella
66) Father ask when the child can go back to school- Rubella – 4 days after appearance of Rash
67) 8 weeks old baby with up to date immunization, which of the vaccines hasn’t been given?
Diptheria, Men C, HIB, Polio, Peturssive, Tetanus
68) Hypoglycaemia patient was given Glucagon? Which organ glucagon act on? Stomach, Pancreas, 165
Liver
69) Which of the pancreatic cells produce glucagon? Alpha, beta, delta,
70) Patient came with lower limb cellulitis – Coagulase negative
71) Malaria which stain is used? Gram, Gemsa, ZN, Iron Haematolysin, Ink stain
72) Patient with renal failure and history of visit of Nigeria, Fever on and off periodically every 72
hours – falciparum, ovale, vivas – I chose falciparum
73) Highest partial pressure of O2- alveolar, ambient air, artery, capillary, venous
74) Child with Mitochondria disease- defect in energy production from glucose oxygenation
75) Patient with dry skin, tachycardia, hot skin, - anticholinergic poisoning
76) Patient with life threatening asthma was given salbutamol infusion – electrolytes abnormality -
hypokalaemia, hypona, hypoCa, HyperK
77) Patient with asthma not relieved after nebulization of salbutamol, Ipratropium bromide was
added on. Mechanism of Ipratropium – anticholinergic muscarinic
78) Causes of low anion gap acidosis – DKA, hypoalbuminaemia, rhabdomyalysis, aspirin
79) Haemetemesis in Heparin – protamine sulphate, Vit K, FFP
80) Coagulation disorder with Prolong PT time- Vii, Xi, Xii
81) Patient with hypocalcaemia but corrected Calcium is normal, why? Hypoalbuminaemia
82) Sedation in antihistamine – can cross blood brain barrier, selective H1 , Selective H2 blocker
83) Common organisms in nursing home-legionella, Chlamydia, Pseudomas, mycoplasma
84) 1 week old neonate with unilateral eye discharge became purulent – Chlamydia, gonorrhea, e-coli
85) Patient with SIADH on drugs- fluoxetine
86) Patient with headache , blurring of vision- pituitary adenoma-acromegaly symtoms – bitemporal
hemianopia
87) Patient wit bitemporal hemianopia site of compression – optic chisma
88) First generation anti psychotic drugs- which receptors – Dopamine
89) Patient presents with extrapyramidal symptoms – due to dopamine receptors
90) Tumour at posterior pituitary . Which of the hormones are produced? Prolactine, ACTH, ADH,
91) Patient with hyperthyroidism – Decreased cholesterol, oligomenorrhoea
92) Hormone produced from hypothalamus- ACTH, CRH, FSH,etc.
93) Patient diagnosed with pheochromocytoma due to increased hormone from – adrenal cortex,
adrenal medulla, Zona G. Zona F, Zona R
94) Patient with Hypertension now having renal failure and HyperK- candesartan, thiazide,
amlodipine
95) Carpopedal spasm few days after thyroidectomy – hypokalaemia, hypocalcaemia, hyper calcaemia
96) 86 years old chronic smoker with COPD- did spirometry test which parameter is increased?
Doesn’t’ remember the stems,
97) Increased Resting cardiac parameter of athlete – HR, SV, Bp
98) Left shift of volume loop – ARDS,asthma, emphysema, pneumothorax
99) Lithium Toxicity – increase permeability of water at collecting duct
100) 100) Ankle injury and MRI shows the ligament injury which attach navicular and
calcaneum- spring ligament
101) 101) The nurse ask you the site for IM to avoid sciatic nerve injury - superior lateral
102) 102) Maturation phase - angiogenesis, granulation, epithelization, collagen formation
103) 103)What is the cell proliferation initial phase of wound healing after homeostasis?
Neutrophils
104) 104)Patient who had splenectomy after RTA is susceptible to infection due to defect in -
neutrophil, T lymphocyte, Mast cell, plasma cell, macrophage
105) 105)Patient diagnosed with mononucleosis - what type of atypical cell?- Lymphocyte,
basophil, neutrophil, monocyte
106) 106)A marathon runner not able to flex the thigh and Xray showed avulsion fracture at
ASIS- Sartorius, pectineus, iliopsoas
166 107) 107)Stab wound at Posterior triangle of the neck and not able to shrug the shoulder and
turn to opposite site- vagus, accessory, glossopharyngeal, etc.
108) 108)A women came with confusion after drinking a plenty of water during aerobic
exercise - due to increased in - ECF tonicity, ECF Volume, ICF tonicity, ICF Volume,
109) 109)defect in downward gaze - trochlear, occulomotor, abducens,
110) 110)A patient came with fever, severe headache, diplopia -cavernous sinus thrombosis -
ethmoid, frontal, maxillary, saggital, sphenoid
111) 111)same scenario diplopia - due to abducens, trochlear, trigerminal, oculomotor
112) 112)Side effect of doxycycline in pregnancy- tooth discolouration
113) 113)A woman became pregnant on OCP after she started new antiepileptics medications
- CBZ, Barbituates, lamotrigenes, valproate
114) 114)Right sided weakness, sensory loss and apraxia stroke - MCA, ACA, Posterior
circulation
115) 115)Spinal cord injury with isolated proprioception loss - dorsal column, anterior
spinothalamic, lateral spnothalamic, corticospinal
116) 116)Fall from height , neck injury with upper limb weakness more than lower limb
weakness- Central cord syndrome, anterior cord, brown sequard
117) 117) pain from acute bladder distension travel through - lateral spinothalmic, anterior
spinothalmic, corticospinal, dorsal column
118) 118)Oesophageal phase in swallowing
119) 119)History of aneurysm and came with headache , bleeding into - subarachnoid,
ventricles, subdural
120) 120)Came in with headache, vertigo, ataxia and nystagmus - posterior stroke in -
Posterior communicating, Posterior cerebral, Posterior inferior Cerebellar
121) 121)Influenza enter the cells through - nucleoprotein, ? amino acid, don’t remember the
rest of the stems
122) 122) Elder woman had Fracture neck of femur - which of the drugs precipitates the
condition? Omeprazole, Simvastatin, Furosemide ,etc. (no steroid , no warfarin , no
methotrexate in the stems)
123) 123)Site of renin production - Afferent arteriole, efferent, Justaglomeurlar apparatus
124) 124)Facial nerve injury but frontalis muscle is spared , nerve supplying orbicularis
oculi - Temporal, Zygomatic, buccal, Mental, Cervical
125) 125)A Patient in a coma after opioid overdose with RR-8 , regained conscious and RR-
18 after naloxone, which of the parameter found to be decreased in repeat ABG? Po2,PCO2,
PH, Bicarbonate,
126) 126)Drug of choice for Whooping cough - Erythromycin
127) 127)How to diagnose whooping cough - nasopharyngeal swap
128) Anions to calculate anion gap- Cloride and bicarbonates, sodium and potassium , sodium and
bicarbonates.
167

FRCEM PRIMARY RECALL – June 2018


Compiled by Dr. Maha Asif

Head n neck

1. Emergency tracheostomy done, wound starts bleeding, which vessel? Jugular vein/
Carotid/brachiocephalic/aortic arc
2. First lower molar infection goes to which lymph node—mandibular lymph nodes
3. Deep wound on parotid, structures damaged? –facial N ,maxillary A
4. Trauma to sella tercica, which bone damaged? … sphenoid
5. Corneal ulcer, pain sensation will travel by which nerve..? ophthalmic of trigeminal
6. Difficulty focusing which muscle involved? .. cilliary
7. Left pupil is dilated when light crosses from right to left eye, which nerve damaged? Oculomotor.
8. Left temporal lobe injury- contralateral homonymous superior quadranopia
9. Ludwig angina is abscess on floor of mouth?? submandibular space
10. 30 yrs old male with dental abscess and sob, the infection has spread to which neck space? ---
Parapharangeal space(also called lateral pharangeal)
11. Laceration on forehead midline, u have to give nerve block. Which nerve will u block with
anesthesia? supratrochlear
12. Head injury scenario, fracture to base of the skull, numbness over upper lip and maxilla. Which
part fractured? foramen rotundum
13. most common aneurysm is anterior communicating artery
14. tragus sensory supply- auriculotemporal nerve
15. Bitemporal hemianipia which artery- posterior cerebral artery

Thorax

16. A child swallows a coin, it lodges in esophagus... structure compressing esophagus at mid
thoracic level... aortic arch
17. external intercostal muscles—elevate the ribs
18. A child appears in emergency department with hematemesis after swallowing a battery, ct shows
it is stuck at T10, which vessel has been damaged and is bleeding? Subclavian/
19. needle piercing what chamber of the heart if gone too far while aspirating for tamponade/ Nerve
that can be damaged when u incise pericardium- phrenic nerve

20. Dermatome at level of nipple… T4


21. aortic diaphragmatic opening---T12
22. A patient has consolidation, and you auscultate which part in RT 5th Intercostal space. ??---
middle lobe
168

23. thoracic inlet syndrome—subclavian


24. Thoracotomy done, bleeding vessel, which one?

Upper Limb

25. DIP joint deformity?? Mallet finger


26. A man fell on open arm, he cannot flex his wrist, Loss of elbow flexion with loss of lat foreman
sensation. Which cord involved... lateral cord
27. Fracture of shaft of humerus—radial nerve damaged
28. What passes above the flexor retinaculum and is outside the carpel tunnel. ----Tendon of flexor
carpi radialis,
29. Flexor tendon sheath continues with??--- The little finger.
30. Superficial wound on extensor surface of proximal IP joint of middle finger, what will be
damaged? Tendons were in option.
31. Trauma to the medial side of arm 5 cm above medial epicondyle. There is a bleeding vessel,
which structure is damaged? ulnar collateral
32. Ant shoulder dislocation, ligament damaged? – glenohumeral
33. Patient cannot abduct his arm from 0-15 degrees. Muscle injured? Supraspinatous
34. Accessory nerve damage.—muscle paralyzed?--- trapezius
35. Loss of sensation over thumb—C6 dermatome
36. Patient cannot laterally rotate- muscle involved- infraspinatous
37. Thenar eminence question- flexor polices bravis

Abdomen-

38. Descending colon blood supply? Inferior mesenteric


39. Umbilicus T10

Lower limb

40. Pain in leg, Patient is unable to flex his foot and big toe. Compartment syndrome. flexor halusis
longus
41. Ankle jerk… L5S1
42. Patient has fractured anterior superior iliac spine during a trauma. Which muscle is affected?
Sartorius
43. Sensory supply to the lateral of sole of foot-- Sural n

CNS

44. 50 year old male hypertensive has a sudden loss of consciousness with quadraplagia, he is
moving eye balls. CT is done, suggestive of bleed in which of the following …thalamus/
pons/basal ganglia
45. Lumber puncture , “give” before entering the csf is what structure—dura/arachnoid/ligamentum
flavum
46. Patient of Bacterial meningitis, csf is cloudy, what differentiates it from viral meningitis...
increased proteins/increased lymphocytes/decreased glucose
169

47. Ptosis, anhydrosis (horner syndrome)


48. Typical features of PICA? Lateral medullary syndrome Posterior inferior cerebellar artery. (PICA)

CRANIAL NERVES

49. Patient with diplopia- which cranial nerve damaged--- oculomotor/ trochlear

PHYSIO

50. Cause of steatorhea – lipase deficiency.


51. henry law
52. canuala used for resus, if radius is doubled, flow will be increased ?--- 16 times
53. Hypersensitivity reaction 1/2/3/4?
54. dead space in a healthy individual- 150
55. residual volume in a healthy individual ?—7ml/kg
56. C1 esterase deficiency ffp
57. Fetal hemoglobin shifts oxygen saturation curve to left becuase? Affinity for oxygen increases
58. Carboxyhb shifts oxygen saturation curve to ….left
59. Haselbach equation requires values of which of the two to be calculated? hco3+co2
60. A-a mismatch... asthma/copd/
61. Pregnant lady which lung volume will decrease? TV
62. sob, Normal A-a ratio.----- copd/asthma/myasthenia gravis/
63. 45 y o male treated for pneumonia, got drowsier after treatment with hyponatremia and
increased urinary sodium, cause? SIADH
64. 13 yr old girl with menhorragia, epistaxis which test will identify the disease. aptt
65. Type-1 pneumocytes in lung have what function?---- secrete surfactant/act as macrophages/gas
exchange/reduce surface tention
66. Enzyme in pancreatic juice? trypsinogin
67. What increases gastric emptying…
68. Chemo receptors that sense co2 changes and effect respiration accordingly are located in? aortic
arch/ carotic body/ j receptors
69. Parietal cells produce? Intrinsic factor
70. Chief cells produce? pepsinogen
71. Type 1 diabetes scenario
72. Glucose filtered in which part of nephron? PCT
73. Acid reflux pathophysiology?
74. Rennin deficiency- hyperkalemia
75. J receptors
76. Graves disease scenario, palpatations due to ?--- T3 /T4/albumin
77. Thyrotoxicosis scenario, medication to be given- B-blockers
78. Scenario of oesophageal varices, asking about other possible location for portosystemic shunt...
umbilicus/ rectum
79. Pacing will activate which area --SA node
80. prolonged QT corresponds to which cardiac phase— phase 3
170

81. Scenario: Wernickys Encaphlopathy, Patient consumes too much alcohol, presented with
vomiting? thiamine deficiency
82. Pulmonary embolism-3 questions
83. Action of parathyroid on DCT-
84. Somatostatin secreted from delta cells
85. Angiography of chest pain patient, occlusion of left circumflential branch, which cardiac area
affected?
86. Ecg shown, v2,v3,v4, which vessel involved based on ecg changes—LEFT anterior descending
87. Ecg changes in hypokalemia –pr increased/
88. Qrs corresponds to which jvp cycle- isovolumetric contraction.
89. Systolic pressure in right ventricle- 15-30mmHg
90. Lead reversal- AVR-AVL
91. Macrocytic anemia- vit b12 deficiancy
92. B12 def--- ilium resection
93. Intentional tremor in Parkinsonism??Essential tremors?? Nigrostriate tract??
94. Scuba diving related question

PHARMA

95. A cardiac patient with acute onset of joint pain, he is a known case of gout and is also on gout.
Which painkiller will you suggest… colchicine, nsaids avoided in cardiac patient?
96. Tetanus prone wound, wound puncture.
97. Tetanus vaccine? Patient fully up-to date with tetanus VC. According to UK protocol, presented
in ED with soiled open wound with fracture of tibia fibula, vaccination options?? No tetanus
required/ tetanus immunoglobulin only/ tetanus img + vaccine etc
98. 20 wks Pregnant lady with Chicken pox exposure—varicella immunoglobulins/varicella vaccine/
check antibody status / no action required
99. Live vaccine? BCG
100. a boy presented after 2 days of dog bite incident & was in high rabies area, anti rabies
regimen?… 5 doses of vaccine in 1 month/vaccine + immunoglobulin
101. A patient presented is jaundiced and is diagnosed to be a case of hemolytic anaemia, cause?
asprin/mefaminic acid/diclo/
102. 20 wks pregnant with herpes, u plan to give acyclovir, MOA of acyclovir?
103. S/e of katemine…hyperventilation/tachycardia/hypotention
104. Child with diarrhea, oral rehydration solution given, what will correct dehydration? Sodium
chloride/sodium citrate/sachrosodium/glucose
105. Case of hyponatrimea, hypertonic saline given to correct this state. How will this effect
intracellular compartment? Efflux of intracellular water/ influx of extracellular water/ no effect
on cell water.
106.likely cause of hypokalemia in patient on polypharmacy : Salbutamol or Bendroflumethazide/
furosemide
107. What type of immunity is it wen u give immunoglobulin. innate
108. Naloxone works on which receptors…… delta/gamma/mu receptors
109. Antibiotic causing cholestatic jaundice? flucloxacillin/ co amoxiclav / cephalosporin
171

110. Ibuprofen causes hematemisis in a patient, mechanism involved? ----mucosal irritation/decrease


in prostaglindins/hcl production/bicarbonate production etc
111. Tendon rupture caused by ---quinalones
112. Patient with gram negative infection. What antibiotic with u give? Cefalosporins/ co-amoxicalv/
trimathoprim
113. Hay fever treatment- anti muscrininic
114. Bicarbonate in which solution?---- Hartman solution
115. Patient develops MI after taking Nsaids which of the following nsaid it is? diclofenac because it is
more common associated with thromboembolic events
116. Antipsychotics acts on which receptor. Dopamine
117. Cholestatic jaundice--co-triamoxiclave
118. For urinary retension, drug given act on which receptor, anti mascarinic, dopamine,
anticholinestrase, acetylecholine.
119. Cyto450 inducer and inhibitor question
120. Precipitation of gout in a patients drug regimen
121. Terrerist attack used nerve gas, Pt. Was given atropine, What remains after given atropine,
/Bronchoconstriction/Increased salivation & lacrimation/Skeletal muscle
relaxation/Bradycardia/tachycardia
122. Dose of Cipro IV in patient with renal problem?? – 200mgBID/ 400mg OD/ 200mg OD etc
123. Erythromycin oral and chloramphenicol drops for neonatal ophthalmoplagia?
124. MOA of colchicine?? Inhibition of microtubules polymerization

MICRO

125. Leptospira gain access through…. Intact skin/droplets/feco-oral rout


126. Man with diarrhea and vomiting then have eye symptoms and urethral discharge, organism
involved?
127. Hep B infection, which one investigation u will order,
128. How does giardiasis attach… oral hooks/sucker disc/
129. Scenario of pneumonia, right middle lobe consolidation, which is the organism…
130. Scenario of Traveler pneumonia
131. Influenza mode of attachment on host...
132. Legionella causes… hyponatremia
133. Pseudomonas inf. Acquired in hospital… heater fans/
134. Chlorhexidine hand wash effective against--- gram positive/gram negative/
135. Coxscacki virus correct answer.
136. Diarrheal illness, ascending paralysis scenario, organism involved? compylobacter jejuni
137. Encapsulated bacteria—ecoli
138. Reactive arthritis which organism- shigella/ compylobactor/ salmonella

PATHOLOGY

139. Supracondyler fracture in a boy, what kind of bone healing will be present after treatment.
Callus/bone/
140. Swelling of ankles... reason/pathophysiology in ccf patient
141. Immunoglobulin mediated immunity is?? innate/natural/
142. Cellulitis patient, process involved??--- capillary wall leakage/fibrin formation
172

143.c3 c4 marker for post streptocoocus glomerulonephritis

EBM

144. You are doing a study based on the outcomes of many previous studies on that topic...What is
this type of study called?
145. You tell a patient he is suffering from xxxx and put him on yyyy. And tell him is a part of a study
and ask him to come for follow up...What is the bias expected in this form of study
146. ..
147. …
148. …
173

Fr
cem pr
imar
yrecal
ls20176thdec

Compi
ledbyDrsar
madal
i.

1.Adr
enal
inedosei
ncar
diacshockact
oacl
s=1:
10k

2.Oesophaguspi
erceatwhi
chdi
aphr
agm l
evel=T10

3.Cont
ent
sofr
ect
ussheat
h=py
rami
dal
is

4.Abdominali
njur
yscenar
io,
cont
ent
sofr
ect
ussheat
h=agai
n
pyr
amidali
s

5.Sl
eab=ANA

6.Thenari
njur
ygl
asspi
eceset
c=opponenspol
ices/
fpl
/fpbet
c

7.Si
adhf
eat
ures=hy
ponat
remi
a

8.Dr
ugcausi
ngsi
adh=f
louxet
ine

9.Waddl
i
nggai
t=supgl
utealner
ve

10. Cr
ushi
njur
y,dr
ugci=suxamet
honi
um

11. Anaphy
laxi
sreact
ionab=I
gE

12. Sur
fact
antf
unct
ion=r
educet
hesur
facet
ensi
on

13. Sur
fact
antf
ormedby=pneumocy
test
ype2

14. Doxy
cycl
ineusei
npr
egnancy=s.
etoot
hdi
scol
our
ati
oni
nbaby

15. Rubel
lamostdanger
ous=i
stt
ri
mest
er(
organogensi
sper
iod)

16. Cr
prel
easedst
imul
atedby=I
L-6

17. Ecgchangesi
nhy
per
kel
eimi
cptbef
oret
hecar
diacar
rest
=tal
l
t
/broadqr
s
174

18. Ret
eroper
it
onealhemat
oma=duodenum (
sadpucker
)

19. Her
niamedi
alt
oepi
gast
ri
cvessel
s=di
recti
ngui
nalher
nia

20. Adenosi
neci=ast
hma

21. L.
phi
ghestsaf
estl
eveli
nadul
ts=l
3-l
4

22. Duodenali
njur
y,d1i
sat=L1

23. Dr
ugt
hati
nct
heact
ionofami
nophy
lenei
ncopd/
ast
hma=FQ:
cipr
o

24. Nonsedat
ingant
ihi
stami
ne=f
enoxi
fi
nadi
ne

25. Ptwi
thepsneckst
if
fness=ant
iemt
icuse=Met
achl
oropr
omi
de

26. Fuer
osomi
deMoa=t
hickascendi
ngl
oop

27. Fuer
osomi
demoa=na/
k/2cl

28. Thi
azi
demoa=dct

29. Car
oti
dsi
nusmassage=atwhi
chl
evel=t
hyr
oidcar
ti
lage,
cri
coi
det
c

30. Fr
ankst
ral
ingl
awi
ninot
ropesuse=mov
eup&l
eftcur
ve

31. Ri
ghtsi
dedoxy
gendi
ssoci
ati
oncur
ve=aci
dosi
slowph

32. Mostef
fect
ivemet
hodofcont
rol
li
nhduodenalph=br
unner
sgl
and-
bi
car
b

33. Fact
orr
egul
ateappet
it
e-sat
iet
y=gher
li
nfr
om epsi
loncel
l?

34. Massi
veasci
tescl
dpt–pl
eur
alef
fusi
on-
spi
romet
ry=

35. Copdonspi
romt
ery=

36. Her
oinusegi
vennal
oxone–abg=i
ncpco2r
espdepr
essi
on

37. Baggi
ngt
oremov
eni
tr
ogen=r
espv
olf
indi
ngt
lc/
frc/
fev
i/f
vc/
RV

38. Thor
aci
cout
letsy
sndr
ome=subcl
avi
anar
ty

39. Per
icar
dit
is=pher
nicner
ve
175

40. At
hel
eteuni
quef
indi
ng=i
ncsr
okev
olume

41. Tet
nusi
gGmosti
ndi
cat
ivef
act
or=par
kinj
urysoi
l

42. Woundi
nleggasf
ormi
ngor
gani
sm =cl
ost
ri
dium per
fri
nges

43. Antt
ri
angl
enecki
njur
y=pl
aty
sma0/
scm/
inf
rahy
oid

44. Weaknessf
lacci
dpar
aly
sisscenar
io=bot
uli
nism

45. Muscledidvi
deddur
ingt
hor
acot
omy=l
ati
ssi
mus
dor
si/
trapezi
us/s.
a

46. Thor
acot
omydonewhi
chner
vei
njur
e=t
hor
acodor
salner
ve,
axi
lar
y
et
c

47. Jawpai
nref
fer
edt
oear=ATN

48. I
nter
nalaudi
tor
ymeat
usner
vei
nj ed=7th+8th
ur

49. Damaget
omast
oidwhi
chner
vei
njur
edj
awr
efl ost=5thner
exl ve

50. Earl
obul
e=n.
s=gr
eat
eraur
icul
arner
ve

51. Or
bicl
uar
isoccul
i=n.
s=

52. Papi
ll
aryl
ightr
efl
ex

53. Safestbetablockerinafi
bwithasthmatic
pt
=atenalol
,car
vi.
prpranal
ol(car
diosel
ect
ive?)

54. Her
pessi
mpl
exencephal
it
isscenar
io=ctf
indi
ng=t
empor
all
obe

55. Expr
essi
vedy
sphasi
acase=br
ocas–f
ronall
obe

56. Opt
icchi
asm –bi
tempor
alhemi
anopi
a

57. Opt
ict
ract–homony
moushemi
anopi
a

58. Antspi
nalar
tysy
ndr
omecase

59. Scenar
ioofl
eftt
emor
al–r
ightnasalv
isi
onl
oss
176

60. Al
dest
ronedef
ici
ency=addsi
on=hy
pona+hy
perk

61. Connssy
ndr
omescenar
io=hy
per
nat
remi
a

62. Ri
ghtmi
ddl
elobeauscul
tat
ion=T4–T5

63. Gout
yptwi
thhear
tfai
lur
e=exacer
bat
ionofgout
=col
cichi
ne

64. Aor
ti
cregur
git
ati
onauscul
tat
ion=r
ight2i
cs

65. Youngptwi
thmeni
ngi
ti
ssi
gnswi
thr
ash=N.
meni
ngi
ti
s

66. Nsai
dwhi
chhav
elessgis,
e=i
bupr
ofen/
indomet
haci
n/di
clo

67. CaseofI
BSnsai
dsuse=b12absor
pti
ondec=b12def

68. Chol
era=i
ncchl
ori
desecr
eti
onchannel
sact
ivi
ty

69. I
ntest
inalf
lor
adi
stur
bance=cl
ost
ri
dium di
ff
ici
le

70. Ti
ghner
vesuppl
yant
-l
at=f
emor
alner
ve

71. Antcompar
tmentf
orear
minj
ury=muscul
ocut
aneousner
ve=c5-
c7

72. Antcompar
tementf
orear
minj
ury=pr
onat
ort
eresnot
heret
c

73. Gl
uteali
njgi
ven–f
ootdr
op=sci
eti
c/commenper
onealner
ve

74. Pneumoni
aol
dagei
nnur
singcar
ehome=or
gani
sm=

75. Hipprost
hesisdi
spl
acementl
igamenti
nvol
ve
=t
ransver
se/i
ll
iol
umber

76. Supr
acl
avi
cul
arf
ossader
mat
ome=C3

77. Si
ckl
ecel
l–ost
eomy
li
ti
s=sal
monel
la

78. Sal
monel
la=f
eaco-
oralr
out
e

79. Si
ckl
ecel
l-
anemi
aduet
o–r
bcsequest
rat
ion

80. Br
eastout
erquadr
entl
.n=antgr
ouporpect
oralgr
oup

81. I
faskedmedi
alquadr
ant=par
ast
ernalnodes
177

82. I
nst
ri
nsi
cmuscl
ehandsi
njur
ylowerbr
achi
alpl
exi
usi
njur
y=c8-
t1

83. Shoul
dercuf
frot
ator
s–i
ntr
otat
ionl
oss+=subscapul
ri
s

84. Lor
ezepam se=r
espdepr
essi
on

85. Mumpsi
sol
ati
onper
iod=5day
s(4-
4)

86. CaseofMumps+decpl
tbl
eedi
ngr
enalf
ail
ure=hsp/
imune
t
hroboct
opni
a

87. Febr
il
echi
ldhavefeverr
ashon9-
10monthv
accinatedall.
fol
lowi
ng
sy
smptomsduet owhich?=mmrvacci
ner
ubel
lasuspectd

88. Postspeenect
omy
=mcor
gani
sm pnemococcal
/h.
inf
luenza

89. Ecol
iinf
ect
ion=f
imbr
iaat
tachment
/hostdef
ense/
flager
ymov
ement

90. S.
ery
sipel
as=s.
pyogenes

91. Hear
tdefectnowI
nfect
iveendocar
dit
is=st
repv
iri
diansal
pha
heamol
yti
c

92. Femoralf
ract
urebl
oodl
ossdecbpt
henmai
ntai
nedbp
=bar
ror
eceptor
s

93. CBF=phy
siol
ogi
cdeci
cpduet
o=i
nit
iali
ncpco2=hy
per
vent
il
ati
on

94. sBP7opostt
ransf
usi
onbpmai
ntai
ned=t
prdec?

95. massi
vePul
monar
yembol
ism =echof
indi
ng=l
eftar
tri
um pr
essur
e
dec?

96. chemor
ecept
orl
ocat
ion=car
oti
dbody
/car
oti
dsi
nus

97. pancr
eas-
exocr
ine-
prot
iendi
gest
ion=t
rypsi
n

98. onl
yendocr
inepancr
eat
icsecr
eti
oni
s=gl
ucagonbyal
phacel
ls

99. f
aci
alner
ve=st
ylomast
oidf
oramen

100.ner
vet
oper
ior
bit
alar
ea=zy
gomat
ic/
inf
raor
bit
al/
buccal
178

101.f
ootdr
opsensat
ionl
oss=l
ater
alcompar
tmentofl
eg

102.sci
ti
cner
vedamage=ext
ensi
onofhi
p+f
lexi
onofknee

103.cushi
ngdi
seases=scenar
io=dmst
estdone=noef
fect

104.medi
alepi
condy
lei
njur
y=muscl
egr
oupi
nvol
vewr
istf
lexor
s/el
bow

105.dashboar
dinj
uryr
ta=pcli
njur
y

106.l
iv
eral
tenzy
meshi
gh=i
ncact
ivi
tyofcy
tosol

107.mi
tochondr
iaf
unct
ion=cel
lul
arr
espi
rat
ion&at
pfor
mat
ion

108.scenar
ioofptonl
mwhwi
thheamt
emesi
s=r
xpr
otami
nesul
phat
e

109.adenosi
neeffectonhear
tmoa=+chr
ontrpi
c/-
chr
ontropi
c/+dr
omot r
opi
c/i
nccor
nor
aybloodperfusi
on

110.i
prat
ropi
um br
omi
dei
nhal
er=r
ecept
ors=muscar
ini
c

111.poi
soi
ngcaseofant
ichi
ler
gict
oxi
drome=achr
ecept
orsbl
ockage

112.hear
tbl
ockcasesomeecgf
indi
ng=sl
owavconduct
ion

113.cutatv
olarwr
istabdpol
ices/
fpb/
o.p

114.t
apewor
minf
est
ati
on=r
x=ni
closami
deorpr
azi
quant
al

115.her
peszost
ergangl
ion=l
atgeni
cul
ate

116.di
lt
iazem s.
e=

117.femalehxofinnat
eimmunity=r
ecurr
entut
iinf
ect
ion=causesur
ine
fl
owdribbl
ingduetostr
uct
uralbybi
rt
hdefect…

118.hy
per
toni
csal
inemoa=i
ntr
acel
lul
ar-
int
rav
ascul
ar/
int
rav
ascul
ar-
I.
C

119.l
owani
ongap=hy
poal
bunemi
a

120.ani
ongaponl
yani
ons=cl
+hco3

121.mucoci
lar
ryescal
atorf
unct
ioncel
ls=ci
li
atedcol
umnarepi
thel
ium
179

122.f
ootdr
op=commenper
onealner
vei
njuy

123.smabl
oodsuppl
y=j
ejuni
um

124.ankl
eref
lex=S1S2

125.hy
ocal
emi
a=v
itddef

126.pt
hdefscenar
io(
deccapt
hdeci
ncph)

127.ni
ppl
eder
mat
ome=T4

128.ppImoa=h/
kat
pasepump

129.r
xofdka=i
mpor
tantEl
ect
rol
yter
epl
acement
=pot
assi
um r
ep

130.t
bcasef
rom i
ndi
a=dxsput
um af
bsmear

131.chi
ldwit
hgast
ri
closs,
vomit
ing,
met
abol
ical
kal
osi
sel
ect
rol
yte
di
stur
bance=duetol
ossofhcl

132.ccbaml
odi
pinese=pedaledema

133.r
ashaf
tert
aki
ngmedi
ntonsi
li
tt
is=amoxi
cil
li
n–pmpduet
o(EBV)

134.toxi
cmegacol
on–dxhi
erschpr
ungdi
seases=my
ent
ri
corauer
bacs
pl
exus

135.ar
trydamagei
nrect
ussheat
h=i
nfepi
gast
icv
essel
-ar
ty

136.moaorNEi
nsept
icshock=v
.c

137.heamost
atsi
sinj
urymostabudandcel
latend=f
ibr
obl
ast/
plat
elet
pl
ug/N/L

138.chr
oni
cgastr
it
isonbiopsypr
edomi nantcel
l
=l
ymphocyt
es/
macrophages/mult
inucleat
edgiantcel
l/
neat
rophi
ll
s

139.gl
ucagonbywhi
chcel
l=al
phacel
lofendocr
inepor
ti
on

140.abdoimi
nalt
rauma–herni
ati
onofintest
inei
ntochestt
hrough=t
7-
t9l
evel
/domesoddiaphr
agm/cent
ralt
endon
180

141.scenar
ioofdxofpheochr
ocy
toma–al
phabl
ockade

142.my st
hani
agravi
ssuspect
edcaseopt
icneut
it
isf
indi
ngs=duet
o
lossofmyel
anati
on

143.woopi
ngcough=nasophar
ngealswab

144.acr
omegal
ycasescenar
io

Note;addev bm 5questi
on,restofotherquest
ioni
fuhaveorcompl ete
themorestems&opt ionsifuhav e,someofquesti
onarepoorrecal
lstry
toaddinthese..ki
ndlyconfi
rmt hekeyalsothi
sisj
ustancoll
ecti
vef or
m
notaconfir
medone…bestofl ucktoall
…mayALLAHsucceedal lofusin
ourgoal
s…. staybl
ess
181

FRCEM Primary examination:


● Anatomy (60 questions)
● Physiology (60 questions)
● Pharmacology (27 questions)
● Microbiology (18 questions)
● Pathology (9 questions)
● Evidence-Based Medicine (6 questions)

# ANATOMY – 60 :
*HEAD AND NECK :
1- platysma – stab wound to anterior triangle of neck – which muscle would be injured (choices were

sternocleidomastoid, scalenous anterior, trapezius, platysma and one other)

2- head injury with fractured internal acoustic meatus, which TWO nerves would be affected – facial

and glossopharyngeal

3- female with mass on anterior 2/3rds of tongue, where is lymph drainage? i chose submental
because

tip of tongue drains there but rest of anterior tongue drains to submandibular, so I am not sure, both

were in the options

4- lower lip numbness, nerve involved? inferior alveolar

5- optic tract lesion, right or left – scenario depicting right homonymous hemianopia

6- headache, increased intracranial pressure symptoms, bitemporal hemianopia ? lesion site – optic

Chiasm

7- mass in optic chiasm, symptomatic, hyperglycemia, likely hormone excess? growth hormone

8- orbital blowout fracture, diplopia on upward gaze. which is entrapped? superior oblique, inferior

oblique, inferior rectus, superior rectus, medial rectus

9- orbital blowout fracture – inferior orbital fissure fracture, which will be damaged?

10- oculomotor nerve – consensual light reflex scenario where light shined in right eye, reflex
present, light moved to left eye but right pupil dilates, lesion?

11- scenario of elderly male involuntary tremors in one hand, gait changes, rigidity in muscles,
expression changes, where is the lesion? substantia nigra

12- neck surgery on thyroid, which nerve is damaged? recurrent laryngeal nerve
182

13- Ant triangle neck injury=platysma0/scm/infrahyoid

14- Muscle divided during thoracotomy=latissimus dorsi/trapezius/s.a

15- Thoracotomy done which nerve injure=thoracodorsal nerve,axillary etc

16- Internal auditory meatus nerve injured=7th+8th

17- Damage to mastoid which nerve injured jaw reflex lost=5thnerve

18- Ear lobule=n.s=greater auricular nerve

19- Orbicluaris occuli=n.s=

20- Jaw pain reffered to ear=ATN

21- Papillary light reflex

22- Optic chiasm –bitemporal hemianopia

23- Optic tract–homonymous hemianopia

24- Scenario of left temoral–right nasal vision loss

25- Expressive dysphasia case=brocas–fronal lobe

26- Supraclavicular fossa dermatome=C 3

27- facial nerve=stylomastoid foramen

28- nerve to periorbital area=zygomatic/infraorbital/buccal

29- mysthania gravis suspected case optic neurtitis findings=due to loss of myelanation

30- Emergency tracheostomy done, wound starts bleeding, which vessel? Jugular vein/

Carotid/brachiocephalic/aortic arc

31- First lower molar infection goes to which lymph node—mandibular lymph nodes

32- Deep wound on parotid, structures damaged? –facial N ,maxillary A

33- Trauma to sella tercica, which bone damaged? … sphenoid

34-Corneal ulcer, pain sensation will travel by which nerve..? ophthalmic of trigeminal

35- Difficulty focusing which muscle involved? .. cilliary

36-Left pupil is dilated when light crosses from right to left eye, which nerve damaged? Oculomotor.

37- Left temporal lobe injury- contralateral homonymous superior quadranopia

38- Ludwig angina is abscess on floor of mouth?? submandibular space

39- 30 yrs old male with dental abscess and sob, the infection has spread to which neck space? ---

Parapharangeal space(also called lateral pharangeal)

40- Laceration on forehead midline, u have to give nerve block. Which nerve will u block with

anesthesia? supratrochlear
183

41- Head injury scenario, fracture to base of the skull, numbness over upper lip and maxilla. Which

part fractured? foramen rotundum

42- most common aneurysm is anterior communicating artery

43- tragus sensory supply- auriculotemporal nerve

44- Bitemporal hemianipia which artery- posterior cerebral artery

45- 50 year old male hypertensive has a sudden loss of consciousness with quadraplagia, he is

moving eye balls. CT is done, suggestive of bleed in which of the following …thalamus/

pons/basal ganglia

46- Ptosis, anhydrosis (horner syndrome)

47- Typical features of PICA? Lateral medullary syndrome Posterior inferior cerebellar artery. (PICA)

48- Patient with diplopia- which cranial nerve damaged--- oculomotor/ trochlear

*CHEST AND SPINAL :


1- hilar lymph adenophathy on CXR? bronchopulmonary? paratracheal? tracheobronchial? superior

thoracic? pulmonary?

2- dentures stuck in trachea at T4/5 level, what causes the constriction? – arch of aorta

3- patient with spinal cord injury, central cord suspected syndrome suspected, clinical signs to be

expected? – proximal musculopathy? upper limb muscles affected worse than lower limb muscles,

4- L.p highest safest level in adults=l3-l4

5- Thoracic outlet syndrome=subclavian arty

6- Pericarditis=phrenic nerve

7- Athelete unique finding=inc sroke volume

8- Ant spinal arty syndrome case

9- Right middle lobe auscultation=T4–T

10- Aortic regurgitation auscultation=right2ic

11- Breast outer quadrent l.n=ant group or pectoral group

12- If asked medial quadrant=parasternal node

13- nipple dermatome=T4

14- A child swallows a coin, it lodges in esophagus... structure compressing esophagus at mid

thoracic level... aortic arch

15 external intercostal muscles—elevate the ribs


184

16- A child appears in emergency department with hematemesis after swallowing a battery, ct
shows it is stuck at T10, which vessel has been damaged and is bleeding? Subclavian/

17- needle piercing what chamber of the heart if gone too far while aspirating for tamponade/ Nerve

that can be damaged when u incise pericardium- phrenic nerve

18- Dermatome at level of nipple… T4

19- aortic diaphragmatic opening---T12

20- A patient has consolidation, and you auscultate which part in RT 5th Intercostal space. ??-middle
lobe

21- thoracic inlet syndrome—subclavian

22- Thoracotomy done, bleeding vessel, which one?

23- Lumber puncture , “give” before entering the csf is what structure—dura/arachnoid/ligamentum

flavum

24- Patient of Bacterial meningitis, csf is cloudy, what differentiates it from viral meningitis...

increased proteins/increased lymphocytes/decreased glucose

*UPPER LIMBS :
1- msuculocutaneaous nerve – stab wound to axilla, causing weakness of elbow flexion and
supination

2- woman presents with right shoulder weakness, difficulty in raising it above her head and also

shoulder contour different – which nerve implicated? accessory (deltoid)

3- froments sign

4- picture of winged scapula, which muscle likely to be affected? – serratus anterior

5- winged chapel scenario in hiker, which nerve is damaged? long thoracic nerve

6- plumber/painter, right elbow lateral condyle tenderness/painful movements of elbow, what

movement will be affected? wrist extension

7- slip and fall on outstretched hand, wrist pain, lunate dislocation, also c/o severe pain in hand,
what is likely to have been injured? – distal radius, radial artery, ulnar nerve, scapholunate
dissociation, radial nerve

14 year old fell from horse, numbness and tingling to medial 2 fingers and ulnar border of hand ?

lesion where?

8- fall on outstretched hand, elbow injury, posterior dislocation of bow, what is likely fractured? –

capetellum, trochlear part, olecranon, radio head, supracondylar region


185

9- volar wrist aspect injury, laceration, possible injury to structures above flexor retinaculum, what
will be injured?

10- slip and fall landed on back/hip, painful movement of hip adduction but hip flexion is normal,
what muscle is injured

11- unable to move below waist, wrist elbow and shoulder movements intact, unable to flex fingers
or grip anything, lesion?

12- fracture base of little finger metacarpal, which muscle injured?

13- Thenar injury glass pieces etc=opponens polices/fpl/fpbetc

14- Ant compartment forearm injury=musculocutaneous nerve=c5-c7

15- Ant compartement forearm injury=pronator teres not heret

16- Instrinsic muscle hands injury lower brachial plexius injury=c8-t1

17- medial epicondyle injury=muscle group involve wrist flexors/elbow

18- Shoulder cuff rotators–int rotation loss+=subscapulri

19- cut at volar wrist abd polices/fpb/o.p

20- DIP joint deformity?? Mallet finger

21- A man fell on open arm, he cannot flex his wrist, Loss of elbow flexion with loss of lat foreman

sensation. Which cord involved... lateral cord

22- Fracture of shaft of humerus—radial nerve damaged

23- What passes above the flexor retinaculum and is outside the carpel tunnel. ----Tendon of flexor

carpi radialis,

24- Flexor tendon sheath continues with??--- The little finger.

25- Superficial wound on extensor surface of proximal IP joint of middle finger, what will be

damaged? Tendons were in option.

26- Trauma to the medial side of arm 5 cm above medial epicondyle. There is a bleeding vessel,

which structure is damaged? ulnar collateral

27- Ant shoulder dislocation, ligament damaged? – glenohumeral

28- Patient cannot abduct his arm from 0-15 degrees. Muscle injured? Supraspinatous

29- Accessory nerve damage.—muscle paralyzed?--- trapezius

30- Loss of sensation over thumb—C6 dermatome

31- Patient cannot laterally rotate- muscle involved- infraspinatous

32- Thenar eminence question- flexor polices bravis


186

*LOWER LIMBS :
1- achilles tendon rupture – ciprofloxacin

2- ankle movement loss – ? location of lesion

3- neck of fibula fracture – common perennial nerve injury


4- valgus deformity of knee joint following football injury i think, ?ligament damaged – medial
collateral

5- hyperextension of knee joint – horse-riding incident, unable to weight bear swollen painful knee
etc, injury to? – anterior cruciate

6- swollen tender knee joint, known recurrent joint problems elderly female, what blood test will be

raised? – rheumatoid factor

7- football injury young male – snapping noise when hip joint moved, avulsion of anterior superior
iliac spine ? muscle involved – sartorius

8- anterior aspect of thigh numbness – lateral cutaneous nerve of thigh

9- fifth metatarsal base fractured – muscle?

10- fibular neck fracture, which muscles is injured?

11- stabbed through adductor canal, injury to what structure? saphenous nerve, femoral nerve,

obturator nerve

12- Tigh nerve supply ant-lat=femoral nerve

13- foot drop sensation loss=lateral compartment of leg

14- dash board injury rta=pcl injur

15- foot drop=commen peroneal nerve inju

16- ankle reflex=S1S

17- Pain in leg, Patient is unable to flex his foot and big toe. Compartment syndrome. flexor halusis

longus

18- Ankle jerk… L5S1

19- Patient has fractured anterior superior iliac spine during a trauma. Which muscle is affected?

Sartorius

20- Sensory supply to the lateral of sole of foot-- Sural n


187

*ABDOMIN & PELVIC :


1- stab 5 cm above iliac crest left lateral abdomen – deepest structure before peritoneum? –

transversus abdominis

2- megacolon found in 7 day old infant, passed meconium at 3 days of both and no bowel
movements since then, which system affected? – myenteric plexus

3- ectopic pregnancy scenario, site of likely pregnancy? – ampulla of fallopian tube

direct inguinal hernia, defect?

4- middle aged male, struck in anterior abdomen with steel girder, fast scan shows intra-abdominal

fluid, which organ would be injured? – spleen injury (other options were kidney, pancreas, colon,

aorta, none likely to be injured than spleen)

5- 48, male, perforation, pneumoperitoneum on CT, likely organ to be perforated? – duodenum,

sigmoid colon, transverse colon, ascending colon, oesophagus

6- elderly patient, abdominal pain, high lactate, very unwell, scenario of ischemic bowel. CT shows

ascending colon affected, what blood supply?- right colic artery

7- FAST scan in patient RTA, hypovolemic shock, where will fluid likely be seen when supine? –
paracolic gutter? para-aortic gutter? between liver and diaphragm, between liver and right kidney,
between spleen and left kidney

8- patient is stabbed in abdomen, diaphragmatic injury at region where oesophagus enters the

diaphragm, what other structure would be injured? vagus, azygos, thoracic duct, aorta, vena cava

9- Oesophagus pierce at which diaphragm level=T10

10- Contents of rectus sheath=pyramidalis

11- Abdominal injury scenario ,contents of rectus sheath=again pyramidalis

12- Waddling gait=sup gluteal nerve

13- Retero peritoneal hematoma=duodenum(sad pucker)

14- Hernia medial to epigastric vessels=direct inguinal hernia

15- Duodenalinjury,d1isat=L1

16- Gluteal inj given–foot drop=scietic/commen peroneal nerve

17- Hip prosthesis displacement ligament involve =transverse/illio lumbe

18- scitic nerve damage=extension of hip+flexion of knee

19- SMA blood supply=jejunium

20- toxic megacolon–dx hierschprung diseases=myentric or auerbacs plexus

21-artry damage in rectus sheath=inf epigastric vessel-arty


188

22- abdoiminal trauma–herniation of intestine into chest through=t7 t9level/ domesod


diaphragm/centraltendo

23- Descending colon blood supply? Inferior mesenteric

24- Umbilicus T10

# PHYSIOLOGY – 60 :
1-loud s1? cause – opening of pulmonary valve, closing of aortic valve, vibration, opening of mitral

valve, closure of tricuspid valve

2- cause of prolonged PR interval

3- cause of prolonged QT interval – clindamycin? (scenario of patient recently using antibiotic for
skin

infection, ECG shows QT prolongation

4- hyperkalemia, ECG changes just before cardiac arrest? Tall, peaked T waves and wide Qrs, Af

5- ECG shown, which vessel involved based on ECG changes?

6- Angiography of chest pain patient, occlusion of left circumflex branch, which cardiac area affected

7- something about starling’s law

8- pre oxygenation for intubation, which lung volume needs to be replaced with o2?

9- MAP calculation formula

10- duodenal/gastric ulcer – likely cause? Gastrin

11- DKA treatment commenced, what will be low as a consequence – potassium

12- dka scenario, what causes drowsiness – sodium

13- osmolality, major contributor? – sodium

14- in patient with addison’s disease, scenario and lab results given what specifically is deficient? –

Mineralocorticoid

14- scenario of tachycardia at 135/min, weight loss, diarrhoea, in a patient with grave’s disease,
what will be lab finding supportive of it? increased T3 hyponatremia, hyperkalemia – hypo
adrenalism

15- scenario of ADH secretion, location? posterior pituitary

16- blood volume? 5 litres

17- recurrent pneumonias in chronic smoker, what is the factor causing him to have recurrent

pneumonias? – decreased mucociliary clearance

18- contact dermatitis scenario? – type 3 hypersensitivity reaction?


189

19- anaphylaxis, which immunoglobulin? – IgM

20- trancutaneous pacing, skeletal muscle contractions of anterior chest wall ? cause – voltage gated

sodium channels opening

21- SLE ab=ANA

22- Siadh features=hyponatremia

23- Drug causing siadh=flouxetine

24- Anaphylaxis reaction ab=IgE

25- Surfactant function=reduce the surface tension

26- Surfactant formed by=pneumocytes type2

27- Ecg changes in hyperkeleimic pt before the cardiac arrest=tall t/broad qrs

28- Adenosine ci=asthma

29- Furosemide Moa=thick ascending loop

30- Furosemide moa=na/k/2cl

31- Thiazide moa=dct

32- Carotid sinus massage=at which level=thyroid cartilage , cricoid etc

30. Frank straling law in inotropes use=move up &left curve

31. Right sided oxygen dissociation curve=acidosis low ph

32. Most effective method of controlling duodenal ph=brunners gland- bicarb

33. Factor regulate appetite-satiety=gherlin from epsilon cell?

34. Massive ascites cld pt–pleural effusion-spirometry=

35. Copdon spiromtery=

36. Heroin use given naloxone–abg=incpco2respdepression

37. Bagging to remove nitrogen=resp vol finding tlc/frc/fevi/fvc/RV

38- Aldosterone deficiency=Addison=hypona+hyperk

39- Conns syndrome scenario=hypernatremia

40- Case of IBS nsaids use=b12absorptiondec=b12de

41- cushing diseases=scenario=dms test done=no effec

42- Femoral fracture blood loss dec bp then maintained bp =barro receptors

43- CBF=physiologic dec icp due to=initial inc pco2=hyperventilation

44- sBP7opost transfusion bp maintained=tprdec?

45- massive Pulmonary embolism =echo finding=left artrium pressure dec?


190

46- chemoreceptor location=carotid body/carotid sinus

47- pancreas-exocrine-protien digestion=trypsin

48- only endocrine pancreatic secretion is=glucagon by alpha cell

49- liver alt enzymes high=inc activity of cytosol

50. Cause of steatorrhea – lipase deficiency.

51. henry law

52. cannula used for resus, if radius is doubled, flow will be increased ?--- 16 times

53. Hypersensitivity reaction 1/2/3/4?

54. dead space in a healthy individual- 150

55. residual volume in a healthy individual ?—7ml/kg

56. C1 esterase deficiency ffp

57. Fetal hemoglobin shifts oxygen saturation curve to left becuase? Affinity for oxygen increases

58. Carboxy hb shifts oxygen saturation curve to ….left

59. Haselbach equation requires values of which of the two to be calculated? hco3+co2

60. A-a mismatch... asthma/copd/

61. Pregnant lady which lung volume will decrease? TV

62. sob, Normal A-a ratio.----- copd/asthma/myasthenia gravis/

63. 45 y o male treated for pneumonia, got drowsier after treatment with hyponatremia and

increased urinary sodium, cause? SIADH

64. 13 yr old girl with menhorragia, epistaxis which test will identify the disease. aptt

65. Type-1 pneumocytes in lung have what function?---- secrete surfactant/act as macrophages/gas

exchange/reduce surface tention

66. Enzyme in pancreatic juice? trypsinogin

67. What increases gastric emptying…

68. Chemo receptors that sense co2 changes and effect respiration accordingly are located in? aortic

arch/ carotic body/ j receptors

69. Parietal cells produce? Intrinsic factor

70. Chief cells produce? pepsinogen

71. Type 1 diabetes scenario

72. Glucose filtered in which part of nephron? PCT

73. Acid reflux pathophysiology?


191

74. Rennin deficiency- hyperkalemia

75. J receptors

76. Graves disease scenario, palpatations due to ?--- T3 /T4/albumin

77. Thyrotoxicosis scenario, medication to be given- B-blockers

78. Scenario of oesophageal varices, asking about other possible location for portosystemic shunt...

umbilicus/ rectum

79. Pacing will activate which area --SA node

80. prolonged QT corresponds to which cardiac phase— phase 3

81. Scenario: Wernickys Encaphlopathy, Patient consumes too much alcohol, presented with

vomiting? thiamine deficiency

82. Pulmonary embolism-3 questions

83. Action of parathyroid on DCT-

84. Somatostatin secreted from delta cells

85. Angiography of chest pain patient, occlusion of left circumflential branch, which cardiac area

affected?

86. Ecg shown, v2,v3,v4, which vessel involved based on ecg changes—LEFT anterior descending

87. Ecg changes in hypokalemia –pr increased/

88. Qrs corresponds to which jvp cycle- isovolumetric contraction.

89. Systolic pressure in right ventricle- 15-30mmHg

90. Lead reversal- AVR-AVL

91. Macrocytic anemia- vit b12 deficiancy

92. B12 def--- ilium resection

93. Intentional tremor in Parkinsonism??Essential tremors?? Nigrostriate tract??

94. Scuba diving related question

*OTHER :

1-hemorrhage, life threatening, what will be the effect on kidneys? – decrease urine production

2- scenario of heart failure, furosemide given, site of action? – loop of henle

3- someone in type 2 resp failure, how will body realise it needs to increase breathing rate? –

Chemoreceptors

4- alcoholic, male, ascites, cause? – portal hypertension


192

5- ascitis in female, lung function tests, which would be increased? FEV1/FVC

6- acute infection with hepatitis B – raised ALT

7- terminal ilium resection, absorption of which will be affected – vitamin c, zinc, proteins,

carbohydrates, vitamin d

8- gout scenario – negatively bifringent crystals on aspirate

9- glucagon secreted from- alpha cells

10- young patient with DKA – which pancreatic islet cell dysfunction? BETA

11- glucagon site of action for gluconeogenesis and glycogenolysis? liver

12- factors that increase gastric emptying – cholecystokinin, histamine, secretin, duodenal
distension,astral distension

13- scenario of scalp laceration, what will cause clotting – conversion of prothrombin to thrombin,

conversion of fibrinogen to fibrin, activation of external clotting cascade

14- pulmonary surfactant function – increase surface tension

15- pulmonary surfactant produced by? – pneumocytes

16- Sickle cell-anemia due to–rbc sequestration

17- 110.ipratropium bromideinhaler=receptors=muscarinic

18- poisoing case of antichilergic toxidrome=ach receptors blockage

19- heart block case some ecg finding=slow av conductio

20- hypertonic saline moa=intracellular-intravascular/intravascular-I.C

21- low anion gap=hypoalbunemia

22- anion gap only anions=cl+hco3

23- mucocilarry escalator function cells=ciliated columnar epithelium

24- hypokalaemia= vit d def

25- pth def scenario(dec capthdecincph

26- ppI moa=h/ k atpase pump

27- rx of dka=important Electrolyte replacement=potassium re

28- child with gastric loss ,vomiting ,metabolic alkalosis electrolyte disturbance=due to loss of hc

29- moa or NE in septic shock=v.c

30- glucagon by which cell=alpha cell of endocrine portion

31- acromegaly case scenario

32- mitochondria function=cellular respiration & atp formation


193

# Pharmacology – 27 :
1-scenario of pancytopenia ? causative drug? – choices included celecoxib and mefenamic acid – i

chose celecoxib

2- lip/peri-oral swelling not improving with adrenaline, which drug implicated mainly as cause? –

ramipril (don’t remember specifically – one of the ‘prils’)

3- anaphylaxis, adrenaline dose? 1:1000

4- scenario of heart failure, furosemide given, site of action? – loop of henle

5- newborn with eye discharge, mummyen under treatment for chlamydia, what is newborn at risk
of?

pneumonia, encephalitis, corneal ulcer, meningitis, and one other option – i chose corneal ulcer

6- digoxin toxocity, when to give digiband? – prolonged seizures, severe bradyarhythmia

7- precipitant of gout in a patient’s drug regimen? – hydrocholothiazide

8-gout treatment, scenario given elderly, heart failure, diabetes, acute gout treatment? – colchicine,

allopurinol, diclofenac, etc etc

9- rationale for steroid use in asthma- reduce bronchial inflammation(duh?)

10- adenosine contraindication (scenario of SVT) – recent severe exacerbation of asthma

11- scenario or warafrinized patient with head injury? reversal with? – FFP

12- digoxin toxicity, worsens – hypokalemia

13- succinylcholine mechanism of action – depolarizing neuromuscular blockage

14- propofol, mechanism of action – GABA receptors

15- cause of warfarin to be potentiated – clarithromycin?

16- cause of pregnancy despite oral contraceptive pills – carbamezepine

17- lithium toxicity – ataxic gait

18- proton pump inhibitor acts on- parietal cell

19- Adrenaline dose in cardiac shock acto acls=1:10k

20- Crush injury ,drug ci=suxamethonium

21- Doxycycline use in pregnancy=s.e tooth discolouration in baby

22- Crp released stimulated by=IL-6

23- Drug that inc the action of aminophylene in copd/asthma=FQ:cipro

24- Non sedating antihistamine=fenoxifinadine

25- Pt with eps necks tiffness=antiemtic use=Metachloropromide


194

26- Safest beta blocker in afib with asthmatic pt=atenolol ,carvi .prpranalol (cardioselective?

27- Gouty pt with heart failure=exacerbation of gout=colcichin

28- Nsaid which have less gis,e=ibuprofen/indomethacin/dicl

29- Lorezepam se=resp depression

30- scenario of pt on lmwh with heamtemesis=rx protamine sulphate

31- adenosine effect on heart moa=+chrontrpic/ chrontropic /+dromotropic /inc cornoray blood
perfusion

32- diltiazem s.e=

33- ccb amlodipine se=pedal edema

34- rash after taking med in tonsilittis=amoxicillin–pmpdueto(EBV)

35- scenario of dx of pheochrocytoma–alpha blockad

36- A cardiac patient with acute onset of joint pain, he is a known case of gout and is also on gout.

Which painkiller will you suggest… colchicine, nsaids avoided in cardiac patient?

37- Tetanus prone wound, wound puncture.

38- Tetanus vaccine? Patient fully up-to date with tetanus VC. According to UK protocol, presented

in ED with soiled open wound with fracture of tibia fibula, vaccination options?? No tetanus

required/ tetanus immunoglobulin only/ tetanus img + vaccine etc

39- 20 wks Pregnant lady with Chicken pox exposure—varicella immunoglobulins/varicella vaccine/

check antibody status / no action required

40- Live vaccine? BCG

41- a boy presented after 2 days of dog bite incident & was in high rabies area, anti rabies

regimen?… 5 doses of vaccine in 1 month/vaccine + immunoglobulin

42- A patient presented is jaundiced and is diagnosed to be a case of hemolytic anaemia, cause?

asprin/mefaminic acid/diclo/

43- 20 wks pregnant with herpes, u plan to give acyclovir, MOA of acyclovir?

44- S/e of katemine…hyperventilation/tachycardia/hypotention

45- Child with diarrhea, oral rehydration solution given, what will correct dehydration? Sodium

chloride/sodium citrate/sachrosodium/glucose

46- Case of hyponatrimea, hypertonic saline given to correct this state. How will this effect

intracellular compartment? Efflux of intracellular water/ influx of extracellular water/ no effect

on cell water.
195

47- likely cause of hypokalemia in patient on polypharmacy : Salbutamol or Bendroflumethazide/

furosemide

48- What type of immunity is it wen u give immunoglobulin. innate

49- Naloxone works on which receptors…… delta/gamma/mu receptors

50- Antibiotic causing cholestatic jaundice? flucloxacillin/ co amoxiclav / cephalosporin

51- Ibuprofen causes hematemisis in a patient, mechanism involved? ----mucosal irritation/decrease

in prostaglindins/hcl production/bicarbonate production etc

52- Tendon rupture caused by ---quinalones

53- Patient with gram negative infection. What antibiotic with u give? Cefalosporins/ co-amoxicalv/

trimathoprim

54- Hay fever treatment- anti muscrininic

55- Bicarbonate in which solution?---- Hartman solution

56- Patient develops MI after taking Nsaids which of the following nsaid it is? diclofenac because it is

more common associated with thromboembolic events

57- Antipsychotics acts on which receptor. Dopamine

58- Cholestatic jaundice--co-triamoxiclave

59- For urinary retension, drug given act on which receptor, anti mascarinic, dopamine,

anticholinestrase, acetylecholine.

60- Cyto450 inducer and inhibitor question

61- Precipitation of gout in a patients drug regimen

62- Terrerist attack used nerve gas, Pt. Was given atropine, What remains after given atropine,

/Bronchoconstriction/Increased salivation & lacrimation/Skeletal muscle

relaxation/Bradycardia/tachycardia

63- Dose of Cipro IV in patient with renal problem?? – 200mgBID/ 400mg OD/ 200mg OD etc

64- Erythromycin oral and chloramphenicol drops for neonatal ophthalmoplagia?

65- MOA of colchicine?? Inhibition of microtubules polymerization

# MICRO – 18 :
1-scenario of impetigo in child, what factor causes its spread? fecal- oral, droplet, intact skin, broken

skin – i chose broken skin.

2- swollen painful knee joint youngish male (35 or so?) no other history, what would you find on
gram
196

stain? gram positive pairs of cocci in clusters, or gram positive cocci in chains, or gram negative rods

or gram positive rods or gram negative intracellular organisms arranged like kidney beans (correct

answer – gonococcal arthritis)

3- picture of a weird target lesion like rash on hand of young male, said to have been acute in onset

over last two days started on limbs now involving trunk etc ? cause – options were HIV, Herpes

simplex, staph aureus (my answer because I thought it was scalded skin syndrome and nothing else

really fit)

4- 3 month old, diagnosed pertussis, most horrible complication? – apneic spells

5- alcoholic male, cough, bloody sputum, fevers, consolidation on xray, microorganism? – klebsiella

6- cholera scenario, profuse watery diarrhoea, mechanism of action? – chloride ions leakage?

7- scenario of steattorhea – defect in exocrine pancreas

8- 35 yo woman with fatigue, icteric sclera, unconjugated bilirubin – hepatitis

9- shingles rash scenario, seen in ED, what would you do to confirm before beginning treatment? –

history and clinical judgement

10- scenario of herpes zoster ophthalmicus, which nerve involved? – trigeminal

11- soiled open wound with open fracture of tibia fibula, treatment options? no tetanus treatment,

tetanus immunoglobulin only, tetanus immunoglobulin and vaccine now, tetanus immunoglobulin

and vaccine now and 1 month later, tetanus vaccine only

12- 8 month old child, fully updated with jabs for age. what has he still not received yet? men a,
men b, hiB, rotavirus, mumps

13- patient has been bitten by someone who is hep b positive, she herself has never received
vaccination for it, management plan? immunoglobulin now, immunoglobulin plus vaccine course
over three months, immunoglobulin plus vaccine course over 6 months

14- child with proven meningitis being intubated in ED, who needs prophylaxis (post-exposure)
clinician intubating, nursing staff in ED, children at party with child, classmates and teachers in
school boy with hemophilia scenario, deficiency? – factor 8

15- patient fully uptodate with tetanus injections according to UK protocol, presents to ED with very
dirty

16- scenario of TB, how to diagnose? – options were CT chest, sputum culture, monteux test,

17- tapeworm infection, how to diagnose? IgE levels raised, eggs in stools

18- diarrheal illness, ascending paralysis scenario, organism implicated? – campylobacter jejuni

19- Rubella most dangerous=1st trimester(organogens is period)

20- Tetnus igG most indicative factor=park injury soil


197

21- Wound in leggas forming organism=clostridium perfringes

22- Weakness flaccid paralysis scenario=botulinism

23- Herpes simplex encephalitis scenario=ctfinding=temporallob

24- Young pt with meningitis signs with rash=N.meningit

25- Cholera=inc chloride secretion channels activity

26- Intestinal flora disturbance=clostridium difficil

27- Pneumonia old age in nursing care home=organism=

28- Sickle cell–osteomylitis=salmonella

29- Salmonella=feaco-oralroute

30 - Case of Mumps +dec plt bleeding renal failure=hsp/immune throboctopnia

31- Febrilechildhavefeverrashon9-10monthvaccinatedall.following Sysmptoms due to which?=mmr


vaccine rubella suspectd

32- Post speenectomy=mcorganism pnemococcal/h.influenza

33- Ecoli infection=fimbria attachment/hostdefense/flagerymovement

34- S.erysipelas=s.pyogenes

35- Heart defect now Infective endocarditis=strepviridians alpha heamolytic

36- tapeworm infestation=rx=niclosamide or praziquantal

37- herpes zoster ganglion=lat geniculate

38- tb case from india=dx sputum afb smear

39- rash after taking med in tonsilittis=amoxicillin–pmp due to(EBV)

40- Mumps isolation period=5days(4-4)

41-rash after taking med in tonsilittis=amoxicillin–pmpdueto(EBV)

42- wooping cough=nasopharngeal swab

43-Leptospira gain access through…. Intact skin/droplets/feco-oral rout

44- Man with diarrhea and vomiting then have eye symptoms and urethral discharge, organism

involved?

45- Hep B infection, which one investigation u will order,

46- How does giardiasis attach… oral hooks/sucker disc/

47- Scenario of pneumonia, right middle lobe consolidation, which is the organism…

48- Scenario of Traveler pneumonia

49- Influenza mode of attachment on host...


198

50- Legionella causes… hyponatremia

51- Pseudomonas inf. Acquired in hospital… heater fans/

52- Chlorhexidine hand wash effective against--- gram positive/gram negative/

53- Coxscacki virus correct answer.

54- Diarrheal illness, ascending paralysis scenario, organism involved? compylobacter jejuni

55- Encapsulated bacteria—ecoli

56- Reactive arthritis which organism- shigella/ compylobactor/ salmonella

# Pathology – 9 :
1-small wound on ankle, swollen acutely inflamed next day – what is the most abundant cell type

present? neutrophils

2- patient with von willibrand disease scenario – what is the dysfunction? – platelet aggregation

3- female hx of innate immunity=recurrent uti infection=causes urineflow dribbling due to structural


by birth defect…
4- heamostatsis injury most abudand cell atend=fibroblast/platelet plug/N/L

5- chronic gastritis on biopsy predominant cell

=lymphocytes/macrophages/multi nucleated giant cell/neutrophils

6- Supra condyler fracture in a boy, what kind of bone healing will be present after treatment.

Callus/bone/

7- Swelling of ankles... reason/pathophysiology in ccf patient

8- Immunoglobulin mediated immunity is?? innate/natural/

9- Cellulitis patient, process involved??--- capillary wall leakage/fibrin formation

10- c3 c4 marker for post streptocoocus glomerulonephritis

You might also like